Psychiatry 2 (11-1 - 28-3)

General

11-1 (102I38) Which of the following does not have dependency issues?

a) Sleeping pills

b) Painkillers

c) Psychostimulants

d) Antipsychotic medications

e) Benzodiazepine anti-anxiety medications

 

The correct answer is:

d) Antipsychotic medications

Explanation:

 

Antipsychotic medications are used to treat symptoms of psychosis such as delusions, hallucinations, and severe agitation. Unlike sleeping pills (a), painkillers (b), psychostimulants (c), and benzodiazepine anti-anxiety medications (e), antipsychotic medications do not typically lead to dependency issues. Patients can use these medications for extended periods without developing a physical dependence. While other medications listed, especially benzodiazepines and opioids (a type of painkiller), are known for their potential for dependency and abuse, antipsychotic drugs are primarily used to manage psychiatric conditions and have a lower risk of leading to substance use disorder.

11-2 (103A12) Which of the following are withdrawal symptoms of sleeping pill dependence? Choose two.

a) Overeating

b) Anxiety

c) Compulsion

d) Delusions of grandeur

e) Seizures

 

The withdrawal symptoms of sleeping pill dependence that may occur are:

b) Anxiety

e) Seizures

Explanation:

  • Anxiety is a common withdrawal symptom associated with the discontinuation of many types of medications, including sleeping pills. Individuals may experience heightened levels of anxiety, restlessness, and nervousness as their body adjusts to the absence of the drug.
  • Seizures can occur as a severe withdrawal symptom, particularly with the discontinuation of benzodiazepines and other sedative-hypnotic sleeping pills. The abrupt cessation of these substances can lead to increased neuronal activity, which may manifest as seizures in some individuals.

11-3 (106D15) Which of the following are related to alcohol dependence? Choose two.

a) Leigh's disease

b) Wernicke's encephalopathy

c) Korsakoff's syndrome

d) Kartagener's syndrome

e) Lambert-Eaton syndrome

 

The correct answers are:

b) Wernicke's encephalopathy

c) Korsakoff's syndrome

Explanation:

  • Wernicke's encephalopathy is an acute neurological condition resulting from thiamine (vitamin B1) deficiency, which is commonly associated with chronic alcoholism. It's characterized by confusion, ataxia, and ophthalmoplegia.
  • Korsakoff's syndrome is a chronic condition that often follows Wernicke's encephalopathy, leading to long-term memory issues. It's also related to thiamine deficiency and is most commonly seen in those with alcohol dependence. It's characterized by significant memory problems, both in forming new memories and recalling existing ones, often accompanied by confabulation (making up stories to fill memory gaps).

Both conditions are part of the spectrum of thiamine deficiency disorders, which are frequently observed in individuals with chronic alcohol dependence due to poor nutritional intake and absorption.

11-4 (106I27) Which of the following cause physical dependence, psychological dependence, and tolerance? Choose two.

a) Cannabis

b) Cocaine

c) Morphine

d) Alcohol

e) Amphetamines

 

The correct answers are:

c) Morphine

d) Alcohol

Explanation:

 

Morphine and alcohol both have the potential to cause physical dependence, psychological dependence, and tolerance. Physical dependence on morphine or alcohol develops as the body becomes accustomed to their presence and begins to require them to function normally. Withdrawal symptoms occur when the substance is abruptly discontinued. Psychological dependence involves a compulsion to use the substance due to its effects on the user's mental state, leading to cravings and compulsive drug-seeking behavior. Tolerance develops as the body adapts to the substance, requiring increasingly larger doses to achieve the same effects. This can lead to escalated use and further entrench dependency. Morphine, as an opioid, is well-documented for its potential to cause these issues, and alcohol's widespread use and the potential for abuse similarly lead to dependence and tolerance in many individuals.

11-5 (107A20) Which combinations of psychoactive substances and their withdrawal symptoms are correct? Choose three.

a) Opium - Miosis (pupil constriction)

b) Alcohol - Sweating

c) Anxiolytics - Insomnia

d) Nicotine - Decreased appetite

e) Methamphetamine - Fatigue

 

The correct answers are:

b) Alcohol - Sweating

c) Anxiolytics - Insomnia

e) Methamphetamine - Fatigue

Explanation:

  • Alcohol - Sweating: Alcohol withdrawal can cause a range of symptoms, including increased heart rate, sweating, and tremors. Sweating is a common withdrawal symptom due to the body's attempt to regulate itself without the presence of alcohol.
  • Anxiolytics - Insomnia: Withdrawal from anxiolytics, especially benzodiazepines, can lead to insomnia. This is because these drugs often act as sedatives, and removing them can lead to a rebound effect, causing difficulty in sleeping.
  • Methamphetamine - Fatigue: Withdrawal from stimulants like methamphetamine often results in fatigue. This is due to the body adjusting to the absence of the stimulant, which previously increased alertness and energy levels.

11-6 (109I2) Which of the following is NOT a withdrawal symptom of alcohol dependence?

a) Hallucinations

b) Excitement

c) Confabulation

d) Tremors

e) Sweating

 

The correct answer is

c) Confabulation

Explanation:

 

Confabulation is not a withdrawal symptom of alcohol dependence. It refers to the fabrication of stories or facts to compensate for memory gaps, often associated with certain brain injuries or conditions like Korsakoff's syndrome, which itself can be related to chronic alcohol misuse. However, it is not a direct withdrawal symptom. Withdrawal symptoms of alcohol dependence typically include hallucinations (a), excitement or agitation (b), tremors (d), and sweating (e), among others. These symptoms occur due to the body's adjustment to the absence of alcohol after a period of heavy and prolonged use.

11-7 (110I29) Which situation suggests abnormal intoxication by alcohol?

a) Complained of nausea after one beer.

b) Became cheerful and talkative after drinking.

c) Started to shout excitedly after drinking.

d) Complained of seeing bugs crawling (hallucinations).

e) Passed out after binge drinking.

 

The correct answer is:

c) Started shouting excitedly after drinking.

Explanation:

 

Options a, b, and e describe phenomena that could occur in healthy individuals as well, referred to as simple intoxication. Option c is correct and corresponds to abnormal intoxication, where the individual's reaction to alcohol consumption is excessively intense or unusual, indicating a potential underlying issue or abnormal sensitivity to alcohol. Option d corresponds to visual hallucinations of small animals, a symptom typically observed during alcohol withdrawal, not during intoxication.

11-8 (114D7) Which of the following is appropriate for the treatment of alcohol dependence?

a) Inpatient treatment is the first choice.

b) Participation in sobriety groups is anonymous by principle.

c) Administer benzodiazepine drugs for withdrawal symptoms.

d) Vitamin D is effective for the prevention of encephalopathy.

e) Administer anti-alcohol drugs in food without informing the patient.

 

The correct answer is

c) Administer benzodiazepine drugs for withdrawal symptoms.

Explanation:

 

Administering benzodiazepine drugs for withdrawal symptoms is a standard approach in treating alcohol dependence. Benzodiazepines help manage the symptoms of alcohol withdrawal, which can include seizures, tremors, anxiety, and agitation. This approach is widely accepted and practiced in medical settings. Other options, such as inpatient treatment (a), may be necessary depending on the severity of the dependence and withdrawal, but it is not always the first choice for every patient. Sobriety groups like Alcoholics Anonymous (b) do operate on principles of anonymity, but this is more about the method of support rather than a direct treatment for alcohol dependence. Vitamin D (d) is not specifically used for the prevention of alcohol-related encephalopathy; instead, thiamine (Vitamin B1) is commonly used to prevent Wernicke-Korsakoff syndrome, a serious brain disorder associated with alcohol abuse. Administering medication without a patient's knowledge or consent (e) is unethical and not an appropriate method of treatment.

11-9 (115A15) Which substances lead to the formation of physical dependence? Choose two.

a) Cannabis

b) Cocaine

c) Morphine

d) Methamphetamine

e) Phenobarbital

 

The correct answers are:

c) Morphine

e) Phenobarbital

Explanation:

  • Morphine, an opioid, and Phenobarbital, a barbiturate, are both known to cause physical dependence. This means that over time, the body becomes accustomed to their effects, and stopping them abruptly can lead to withdrawal symptoms.
  • Morphine binds to opioid receptors in the brain, reducing the perception of pain but also leading to tolerance and physical dependence as the body adapts to its presence.
  • Phenobarbital affects the GABA system in the brain, which is involved in regulating neuronal excitability. Regular use alters the brain's chemistry, leading to dependence.
  • Cannabis (a), Cocaine (b), and Methamphetamine (d) can lead to psychological dependence, where there is a strong desire to keep using, but physical dependence, characterized by withdrawal symptoms upon cessation, is more strongly associated with opioids like Morphine and depressants like Phenobarbital.

11-10 (115F6) Which of the following substances causes psychiatric symptoms that demonstrate a reverse tolerance phenomenon?

a) LSD

b) Cannabis

c) Alcohol

d) Diazepam

e) Methamphetamine

 

The correct answer is:

e) Methamphetamine 

Explanation:

 

Methamphetamine can cause a phenomenon known as reverse tolerance or sensitization, where increased effects are observed with repeated use even at lower doses. This is particularly true for its psychostimulant effects and the potential to cause psychosis-like symptoms. Unlike substances that might lead to a straightforward tolerance, where more of the substance is required over time to achieve the same effect, methamphetamine can enhance susceptibility to its neurotoxic and psychological effects with continued use, leading to severe psychiatric symptoms even at doses that previously produced milder effects.

Clinical

12- 1 (101G59) A 27-year-old woman was brought to the clinic by her father, who reported that she was "saying things that don't make sense." Multiple injection marks are observed on her left forearm. She started using drugs half a year ago, which her live-in partner bought from the entertainment district. She reports injecting the drug into her veins herself. Initially, she experienced euphoria, a lack of fatigue, and felt her thoughts were clearer, enjoying the pleasurable effects. However, a month ago, she began hearing voices threatening to kill her and has been obsessed with the delusion that she is always being targeted by gangsters.

Which of the following statements are correct? Choose two.

a) Discontinuation of the drug leads to delirium.

b) Flashbacks do not occur.

c) Antipsychotic medication is effective.

d) Mental dependence does not occur.

e) Test for infectious diseases.

 

The correct answers are:

c) Antipsychotic medication is effective.

e) Test for infectious diseases.

Explanation:

  • c) Given the patient is experiencing hallucinations and delusions, which are psychotic symptoms, antipsychotic medications can be effective in managing these symptoms. These drugs help to reduce or eliminate the hallucinations and delusions that can occur as a result of substance abuse, especially substances with psychoactive effects.
  • e) Testing for infectious diseases is critical in individuals with a history of intravenous drug use. Sharing needles or using unsterile equipment increases the risk of transmitting blood-borne infections such as HIV, hepatitis B, and hepatitis C. Therefore, it's important to screen for these conditions to provide appropriate treatment or management if needed.

The other options:

  • a) While delirium can occur upon the discontinuation of certain substances, it's not a universal outcome and depends on the specific substance used. Delirium is more commonly associated with alcohol withdrawal.
  • b) Flashbacks can occur with substances that have hallucinogenic properties, but the statement that they do not occur is too absolute without specifying the substance.
  • d) Mental dependence is a common consequence of substance use, especially with substances that alter mood and perception. The pleasurable effects described by the patient can lead to a psychological desire to continue using the substance to replicate those effects.

12-2 (102I78) A 28-year-old woman was admitted to the internal medicine department for the treatment of hepatitis B. On the day of admission, she became excited and started shouting, leading to her transfer to the psychiatry department. She talks fearfully about being wiretapped, surveilled, and being killed. She has previously experienced a similar condition due to methamphetamine abuse, but she had not used methamphetamine for the past two years. This condition rapidly developed upon her current admission. Her consciousness is clear.

Which of the following is the most likely?

a) Acute intoxication state of methamphetamine

b) Physical dependence state of methamphetamine

c) Withdrawal symptoms of methamphetamine dependence

d) Rebound phenomenon of stopping methamphetamine use

e) Flashback of methamphetamine psychosis

 

The correct answer is:

e) Flashback of methamphetamine psychosis

Explanation:

 

This patient exhibits paranoid delusions and fear of being harmed, symptoms that align with methamphetamine psychosis. The fact that she had not used methamphetamine for the past two years and suddenly experienced these symptoms upon admission suggests a flashback rather than acute intoxication or withdrawal symptoms. Flashbacks are involuntary recurrences of emotional and psychological experiences associated with previous drug use, occurring without recent drug intake. This differentiates the situation from acute intoxication (a), which would require recent drug use, and from physical dependence (b) or withdrawal symptoms (c), which arise from cessation after continuous use. The scenario also does not describe a rebound phenomenon (d), which typically involves the return of symptoms that were previously controlled by the drug.

12-3 (112A63) A 57-year-old man has been hospitalized for loss of appetite and liver dysfunction. He has been drinking three cups of sake every day since his twenties without it affecting his work. However, his alcohol consumption increased to more than five cups daily three years ago. A week ago, he started feeling general fatigue and began taking time off work. Despite this, he continued drinking until he could no longer eat due to significant loss of appetite three days before visiting the outpatient clinic, where blood tests revealed liver dysfunction, leading to his hospitalization. Since being admitted, he has experienced insomnia at night, restlessness, and pacing from the second day of hospitalization. At night, he became agitated, claiming "animals are crawling on the walls," with noticeable coarse tremors in his hands and significant sweating, appearing confused about his hospitalization. No abnormalities were found in a head CT scan at the time of admission.

Which medication should be administered first? Select two.

a) Antabuse (Disulfiram)

b) Diazepam

c) Vitamin B group

d) Imipramine

e) Levodopa (L-dopa)

 

The appropriate medications to administer first are:

b) Diazepam

c) Vitamin B group

Explanation:

  • Diazepam: Benzodiazepines like diazepam are effective in managing withdrawal symptoms, including agitation, tremors, and insomnia, commonly seen in alcohol withdrawal. They help stabilize the central nervous system and reduce the risk of severe withdrawal complications such as seizures.
  • Vitamin B group: Thiamine, a Vitamin B1, is crucial for individuals with alcohol use disorder to prevent Wernicke-Korsakoff syndrome, a serious brain disorder resulting from thiamine deficiency. Administration of the Vitamin B group is essential for nutritional replenishment and neurological protection.

12-4 (116F50) A 33-year-old man has had an increase in absenteeism for the past three months and was pointed out at work for smelling of alcohol. After being advised by the occupational physician, he visited a psychiatrist and was diagnosed with alcohol dependence. During the consultation, he stated, "I've been drinking more because work is busy and I feel down. I was told I drink too much, so I stopped drinking three days ago." He exhibits hand tremors and sweating.

Which of the following is correct about this patient?

a) Physical dependence is observed.

b) The doctor should recommend resignation.

c) The likelihood of concurrent depression is low.

d) The first choice of treatment is pharmacotherapy.

e) He is likely to actively seek treatment on his own.

 

The correct answer is:

a) Physical dependence is observed.

Explanation:

 

The patient's increase in absenteeism, being pointed out for smelling of alcohol at work, and the symptoms of hand tremors and sweating upon cessation of alcohol consumption are indicative of physical dependence on alcohol. These symptoms are withdrawal symptoms that occur when a body that has become dependent on alcohol does not receive it. This showcases the physical aspect of dependence, where the body has adjusted to the presence of alcohol and experiences physiological stress when alcohol is not consumed. The patient's attempt to quit drinking and the resulting withdrawal symptoms, such as tremors and sweating, directly point to physical dependence.

General

13-1 (109B11) Which of the following is characteristic of organic mental disorders?

a) Perseveration

b) Flight of ideas

c) Thought blocking

d) Autistic thinking

e) Emotional blunting

 

The correct answer is:

a) Perseveration

Explanation:

 

Perseveration, the repetitive and continuous repetition of a particular response despite the absence or cessation of a stimulus, is characteristic of organic mental disorders. This can occur in conditions where there's brain damage or dysfunction, often observed in neurodegenerative diseases, brain injuries, or other conditions affecting brain function. It reflects a deficit in executive functions, which are responsible for controlling and managing cognitive processes, including working memory, flexible thinking, and self-control. Organic mental disorders affect the brain physically and chemically, leading to symptoms like perseveration, where the patient repeats words, phrases, or actions uncontrollably.

Clinical

General

15-1 (103I10) Which of the following is observed in anorexia nervosa? Select three.

a) Bradycardia

b) Anemia

c) High T3 levels

d) High LH/FSH levels

e) Hypoalbuminemia

 

The correct answers are:

a) Bradycardia

b) Anemia

e) Hypoalbuminemia

Explanation:

  • a) Bradycardia: Anorexia nervosa often leads to a slowed heart rate or bradycardia as the body attempts to conserve energy due to the lack of nutritional intake. This is a common physiological response to starvation or severe caloric restriction.
  • b) Anemia: Anemia can develop in individuals with anorexia nervosa due to inadequate intake of essential nutrients, including iron, vitamin B12, and folate, which are necessary for the production of red blood cells.
  • e) Hypoalbuminemia: This condition, characterized by low levels of albumin in the blood, can occur in anorexia nervosa due to poor nutritional intake. Albumin is a protein produced by the liver, and its levels can decrease in malnutrition, contributing to various complications including edema.
  • c) High T3 levels are not typically associated with anorexia nervosa. In fact, individuals with this disorder may have low T3 levels as the body reduces its metabolic rate in response to starvation.
  • d) High LH/FSH levels are not characteristic of anorexia nervosa. Hormonal imbalances in anorexia nervosa often lead to low levels of reproductive hormones, including LH and FSH, contributing to amenorrhea (absence of menstruation) in women.

15-2 (104D19) Which of the following is correct about bulimia nervosa (binge eating disorder)?

a) Body weight exceeds the standard.

b) Often accompanied by amenorrhea.

c) Engages in binge eating but not fasting.

d) In recent years, it has a lower incidence rate than anorexia nervosa.

e) Frequently associated with electrolyte abnormalities and metabolic alkalosis.

 

The correct answer is:

e) Frequently associated with electrolyte abnormalities and metabolic alkalosis.

Explanation:

 

Bulimia nervosa, also known as binge eating disorder, involves episodes of binge eating followed by inappropriate compensatory behaviors to prevent weight gain, such as self-induced vomiting, misuse of laxatives, diuretics, or other medications, fasting, or excessive exercise. This cycle can lead to significant electrolyte imbalances due to the purging behavior, leading to conditions like hypokalemia (low potassium levels), which can cause metabolic alkalosis. The other options, such as body weight consistently exceeding the standard (a) or being often accompanied by amenorrhea (b), are more typically associated with anorexia nervosa. The statement that individuals with bulimia nervosa engage in binge eating but not fasting (c) partially describes their behavior but does not capture the compensatory actions they take. The claim that bulimia nervosa has become less frequent in recent years than anorexia nervosa (d) is not accurate; the prevalence rates of eating disorders can vary, but bulimia nervosa remains a common eating disorder.

15-3 (108D19) Which of the following is correct about anorexia nervosa? Select three.

a) Many exhibit compulsive behaviors.

b) Tachycardia is common even at rest.

c) Does not include episodes of overeating and vomiting.

d) Comorbidity with depression is frequently observed.

e) In Japan, about 10% of patients die within 10 years.

 

The correct answers are:

a) Many exhibit compulsive behaviors.

d) Comorbidity with depression is frequently observed.

e) In Japan, about 10% of patients die within 10 years.

Explanation:

  • a) Many individuals with anorexia nervosa exhibit compulsive behaviors related to food, eating, and exercise, which is consistent with the emphasis on control over body weight and shape seen in this disorder.
  • d) Comorbidity with depression is frequently observed in anorexia nervosa. The stress of living with this eating disorder, along with biological factors, can contribute to depressive symptoms.
  • e) Anorexia nervosa has a significant mortality rate, and in Japan, about 10% of patients die within 10 years of diagnosis. The high mortality rate is associated with medical complications of the disorder and an increased risk of suicide.

b) and c) are incorrect because:

  • Tachycardia is less common; bradycardia is more typical due to the body's attempt to conserve energy under starvation conditions.
  • Anorexia nervosa can include episodes of overeating and vomiting, especially in the binge/purge subtype.

Clinical

16-1 (101G5) A 16-year-old girl came to the clinic with weight loss as her main complaint. She is an identical twin sister. She had good grades in elementary and middle school and was the tennis team captain. Upon entering high school, she and her sister started dieting. As her sister's academic performance improved, she received more praise from their mother and drifted away from dieting. However, the patient continued with the diet and a year later, her weight dropped to 32 kg at a height of 158 cm. She does not engage in comfort eating or self-induced vomiting.

Which of the following is the least likely condition?

a) Distorted body image

b) Refusal to mature

c) Conflict regarding gender identity

d) Conflict with mother

e) Conflict with sister

 

The least likely condition in this scenario is:

c) Conflict regarding gender identity

Explanation:

 

The presented case describes a young girl with significant weight loss due to continued dieting, a common feature in individuals with anorexia nervosa, characterized by a distorted body image (a) and potentially issues related to maturation (b). The absence of comfort eating or self-induced vomiting indicates a focused and restrictive type of eating disorder, where the primary concern is not about eating behaviors per se but rather about body image and control over weight.

 

The conflicts with the mother (d) and sister (e) could be relevant in the context of seeking approval or competing for attention, which are not uncommon in families with high achievers or multiple children. However, there is no mention of any issues related to gender identity or discomfort with her gender, making option c) Conflict regarding gender identity, the least likely condition associated with her current state. Eating disorders, including anorexia nervosa, often involve complex psychosocial factors, but they do not inherently involve conflicts about gender identity unless specifically mentioned or indicated by the patient's experiences or statements.

16-2 (104A39) A 20-year-old woman visited the clinic with complaints of constipation, dry skin, and amenorrhea for the past year. She has no significant past medical history. After graduating high school, she exhibited abnormal eating behaviors, leading to extreme weight loss. She lives in a family of four with her parents and brother and has strong conflicts with her mother, resulting in minimal daily conversations. Her consciousness is clear. Her height is 162 cm and weight is 38.5 kg. Her body temperature is 35.8°C, pulse 56 beats per minute, regular, and blood pressure 92/56 mmHg.

Which condition is observed in this disease?

a) Lack of disease awareness

b) Loss of pubic hair

c) Skin atrophy

d) Decreased activity

e) Skin pigmentation

 

The correct answer is:

a) Lack of disease awareness

Explanation:

 

In the context provided, where a 20-year-old woman experiences constipation, skin dryness, and amenorrhea for a year following a significant weight loss that began after high school graduation, the lack of disease awareness (a) is a key characteristic of anorexia nervosa. Patients with anorexia nervosa often do not recognize the severity of their condition, attributing their physical symptoms to other causes rather than acknowledging the impact of their eating disorder. This denial or lack of insight into their health status is a critical barrier to seeking and accepting help. It's important for healthcare professionals to approach such patients with understanding and to gently guide them toward recognizing their illness and the need for treatment.

16-3 (105D55) A 14-year-old girl came for further evaluation after a school health checkup indicated hypercholesterolemia. She began dieting and reducing her food intake with a friend a year and a half ago and started strength training six months ago. Recently, she has been feeling very fatigued. Her weight before starting the diet was 43 kg. She is alert but has a sparse expression. Her height is 151 cm (-0.9 SD), and weight is 27 kg (-2.8 SD). Her body temperature is 35.6°C, pulse is 44/min, regular, and blood pressure is 110/92 mmHg. Heart and lung sounds are normal. The abdomen is flat and soft. The skin is notably dry, and there is a cold sensation in the limbs. Blood biochemistry shows: total cholesterol 268 mg/dL, triglycerides 82 mg/dL, TSH 4.6 μU/mL (standard 0.2-4.0), FT3 1.8 pg/mL (standard 2.5-4.5), and FT4 0.8 ng/mL (standard 0.8-2.2). Mild cerebral atrophy is noted on head MRI.

Which symptoms are observed in this disease? Choose three.

a) Amenorrhea

b) Myxedema

c) Binge eating

d) Self-induced vomiting

e) Achilles tendon thickening

 

The correct answers are:

a) Amenorrhea

c) Binge eating

d) Self-induced vomiting

Explanation:

  • a) Amenorrhea: It's a common symptom in individuals with significant weight loss and nutritional deficiencies, reflecting hormonal imbalances and the body's response to starvation.
  • Binge eating: This could be seen in the context of eating disorders, where periods of extreme restriction are followed by episodes of loss of control over eating.
  • d) Self-induced vomiting: This behavior is associated with attempts to control weight after eating, which can be a part of the symptomatology in eating disorders, particularly when the individual is engaging in cycles of restriction and attempts to mitigate the effects of any perceived overeating.

16-4 (112D71) A 17-year-old girl has been hospitalized for malnutrition. About a year and a half ago, upon entering high school, she began reducing her food intake to lose weight. Later, in addition to dietary restrictions, she started jogging 3 km every morning from 6 am. Four months ago, her menstrual periods stopped, and a month ago, she began to experience significant fatigue. There's no self-induced vomiting or abuse of laxatives. Even after hospitalization, her food intake remains low as she expresses a desire not to gain weight. She had no particular adaptation problems in elementary or middle school and has been academically successful. Her height is 158 cm, and she weighs 30 kg.

Which are the most likely symptoms to be observed in this patient? Choose two.

a) Bradycardia

b) Hypothermia

c) Loss of pubic hair

d) Hyperkalemia

e) Hyperprolactinemia

 

The most likely symptoms to be observed in this patient are:

a) Bradycardia

b) Hypothermia

Explanation:

  • a) Bradycardia: Malnutrition, especially in the context of an eating disorder like anorexia nervosa, can lead to a slowed heart rate as the body tries to conserve energy. This is a common finding in severely underweight patients.
  • b) Hypothermia: Similar to bradycardia, hypothermia in this patient can result from malnutrition and the body's decreased metabolic rate. The body is less able to maintain its temperature due to a lack of insulating body fat and reduced energy to generate heat.
  • The loss of pubic hair (c) could also be a symptom due to hormonal changes and malnutrition, but bradycardia and hypothermia are more directly related to the critical physiological adaptations to prolonged starvation and severe weight loss. Hyperkalemia (d) is less likely without evidence of renal dysfunction or specific medication use that could cause such an imbalance. Hyperprolactinemia (e) could occur due to stress or other factors but is not as directly connected to the described symptoms as bradycardia and hypothermia, which are immediate physiological responses to the state of starvation and severe malnutrition.

16-5 (113F44) A 22-year-old woman was brought in by ambulance due to eating disorders and muscle weakness. She started restricting her diet at 18, following a breakup, and has been under psychiatric care as her eating and weight loss escalated. Her condition worsened to the point where she became immobile at home, prompting the emergency call. She has not had her menstrual period for three years. On admission, she was alert with a height of 152 cm, weight of 26 kg, body temperature of 35.1°C, heart rate of 48 bpm, and blood pressure of 80/52 mmHg. Lanugo hair growth was observed on her forearms. Urine analysis showed no protein or sugar but positive for ketone bodies. Blood tests showed RBCs at 4.08 million, Hb at 11.0 g/dL, Hct at 38%, WBCs at 3,300, platelets at 80,000, AST at 28 U/L, ALT at 16 U/L, BUN at 12 mg/dL, creatinine at 0.6 mg/dL, Na at 135 mEq/L, K at 3.0 mEq/L, Cl at 94 mEq/L, blood sugar at 45 mg/dL, HbA1c at 4.4% (normal range 4.6-6.2), TSH at 2.8 μU/mL (normal range 0.5-5.0), FT3 at 1.8 pg/mL (normal range 2.3-4.3), and FT4 at 0.9 ng/dL (normal range 0.9-1.7). After starting intravenous infusion with glucose, she developed respiratory distress, impaired consciousness (JCS II-20), and generalized edema on the second day of admission, with blood tests showing AST at 539 U/L and ALT at 654 U/L.

Which of the following actions is NOT appropriate?

a) Administering phosphorus.

b) Performing an electrocardiogram.

c) Measuring trace elements.

d) Administering vitamin B1.

e) Administering thyroid hormones.

 

The correct answer is:

e) Administering thyroid hormones.

Explanation:

 

This patient's presentation, including her sudden onset of respiratory distress, altered consciousness, and generalized edema after starting intravenous glucose infusion, suggests refeeding syndrome. Refeeding syndrome is characterized by severe electrolyte and fluid shifts associated with metabolic abnormalities in malnourished patients undergoing refeeding, whether enterally, parenterally, or orally. The primary concern in refeeding syndrome is the deficiency of phosphorus, potassium, and magnesium, which can cause severe complications, including cardiac failure, respiratory failure, and seizures.

 

  • Administering phosphorus (a), performing an electrocardiogram (b), and measuring trace elements (c) are appropriate actions in this context to monitor and manage potential complications of refeeding syndrome. Administering vitamin B1 (d) is crucial to prevent Wernicke's encephalopathy, a serious neurological disorder caused by thiamine (vitamin B1) deficiency, which can be precipitated by refeeding.
  • Administering thyroid hormones (e) is not appropriate in this context without specific evidence of hypothyroidism that requires treatment. The patient's TSH and free thyroid hormone levels (FT3 and FT4) are within or near the normal range, indicating that her thyroid function is not the immediate concern. The focus should instead be on managing the acute risks associated with refeeding syndrome and supporting her nutritional and electrolyte balance carefully.

16-6 (114F55) A 17-year-old girl came to the clinic complaining of weight loss. She started intentionally reducing her food intake two years ago and was noted to be underweight during a routine school health check-up. She was advised to seek medical consultation but did not follow through. Her weight continued to decrease, prompting her worried mother to accompany her to the clinic. Her height is 150cm, and her weight is 27kg. Her temperature is 36.1°C, pulse 52 beats per minute, regular, and blood pressure 90/50mmHg. Increased lanugo hair on her forearms and back, and mild pitting edema on her lower legs were observed.

Which of the following is most likely observed in this patient?

a) Elevated GH (Growth Hormone)

b) Normal menstrual cycle

c) Elevated LH/FSH ratio

d) Low cortisol levels

e) Elevated Free Triiodothyronine (FT3)

 

The correct answer is:

a) Elevated GH (Growth Hormone)

Explanation:

  • GH (Growth Hormone) being high (a) can indeed occur in the context of anorexia nervosa. Growth hormone levels may rise as part of the body's effort to conserve energy and maintain glucose levels in the face of starvation. Elevated GH levels can be part of the body's complex hormonal response to the stress of malnutrition and the need to mobilize energy reserves.

  • The expectation of a normal menstrual cycle (b) would be incorrect in this context, as significant weight loss typically leads to amenorrhea due to hormonal imbalances resulting from malnutrition.

  • An elevated LH/FSH ratio (c) is not characteristic of anorexia nervosa; it's more indicative of conditions like polycystic ovary syndrome (PCOS).

  • Low cortisol levels (d) would be an unusual finding in anorexia nervosa since cortisol levels often increase in response to the physiological stress of starvation.

  • Elevated FT3 (e) would be contrary to the typical findings in someone with severe weight loss due to anorexia nervosa, as metabolic adaptations often lead to decreased levels of FT3 in a condition known as non-thyroidal illness syndrome or euthyroid sick syndrome.

General

17-1 (105I17) Which combination of sleep disorder and medication is incorrect?

a) Narcolepsy - Methylphenidate

b) Sleep Apnea Syndrome - Diazepam

c) Circadian Rhythm Sleep-Wake Disorder - Melatonin

d) Restless Legs Syndrome - Pramipexole

e) REM Sleep Behavior Disorder - Clonazepam

 

The incorrect combination is:

b) Sleep Apnea Syndrome - Diazepam

Explanation:

 

Diazepam, a benzodiazepine, is not recommended for treating Sleep Apnea Syndrome because it can worsen the condition. Benzodiazepines relax the muscles, including those in the throat, which can lead to increased airway obstruction and exacerbation of sleep apnea symptoms. Treatments for sleep apnea typically involve the use of continuous positive airway pressure (CPAP) devices, lifestyle changes, or surgery, rather than benzodiazepines.

17-2 (106I25) Which is most likely to occur 2-3 hours after falling asleep?

a) Delirium

b) Nightmare Disorder

c) Sleepwalking Disorder

d) Narcolepsy

e) Circadian Rhythm Sleep-Wake Disorder

 

The correct answer is:

c) Sleepwalking Disorder

Explanation:

 

Sleepwalking typically occurs during the first third of the sleep period, which often corresponds to the deep non-REM (NREM) sleep stages. This stage is most prevalent during the first part of the night, approximately 2-3 hours after falling asleep. During these stages, the brain is in a deep sleep, but the ability to perform motor functions such as walking is still active, leading to sleepwalking episodes. Unlike delirium (a), nightmare disorder (b), narcolepsy (d), and circadian rhythm sleep-wake disorder (e), sleepwalking specifically involves complex behaviors during sleep, including walking, which are initiated during these deep stages of sleep.

17-3 (108I2) Which statement is correct about Restless Legs Syndrome (RLS)?

a) It is closely related to REM sleep.

b) Dopamine-blocking drugs are effective.

c) The number of patients decreases with age.

d) The abnormal sensations in the legs do not improve with movement.

e) Involuntary movements, such as of the ankle, are observed after falling asleep.

 

The correct answer is:

e) Involuntary movements, such as of the ankle, are observed after falling asleep.

Explanation:

  

Restless Legs Syndrome (RLS) is characterized by an irresistible urge to move the legs, typically accompanied by uncomfortable sensations. These symptoms usually worsen during periods of rest or inactivity, particularly in the evenings and at night, and can significantly interfere with sleep. The key feature of RLS is that the symptoms are relieved by movement, such as walking or stretching.

  • a) It is not closely related to REM sleep; RLS symptoms typically occur during periods of rest and can disrupt the process of falling asleep.
  • b) Dopamine-blocking drugs are not effective for RLS; in fact, medications that increase dopamine levels, such as dopaminergic agents, are commonly used to treat RLS.
  • c) The prevalence of RLS does not necessarily decrease with age; it can affect individuals of any age, though it may be more common or recognized more often in older adults.
  • d) The statement that the abnormal sensations in the legs do not improve with movement is incorrect; a hallmark of RLS is improvement of symptoms with movement.
  • e) While involuntary movements are not specific to RLS and do not exclusively occur after falling asleep, individuals with RLS may experience periodic limb movements during sleep, which are repetitive movements typically involving the lower limbs. However, the characterization of involuntary movements, such as of the ankle, as a defining feature after falling asleep, might not be precise. The primary characteristic of RLS involves discomfort and the urge to move the legs, primarily during periods of rest and inactivity, with symptom relief upon movement.

17-4 (111B38) Which of the following statements about human circadian rhythms are correct? Choose two.

a) They change with exposure to high-intensity light.

b) The daily variation in core body temperature is an indicator.

c) The cycle in adults is approximately 27 hours.

d) With aging, they shift later, making sleep times later.

e) Adapting to jet lag is easier by advancing the cycle than by delaying it.

 

The correct answers are:

a) They change with exposure to high-intensity light.

b) The daily variation in core body temperature is an indicator.

Explanation:

  • a) Circadian rhythms change with exposure to high-intensity light. This is because light is a primary cue (or "zeitgeber") that helps to reset our internal clocks daily. Exposure to light, especially in the morning, can advance our circadian rhythms, making us feel more awake and alert during the day.
  • b) The daily variation in core body temperature is indeed an indicator of our circadian rhythms. Typically, our core body temperature drops during the night, reaching its lowest point in the early morning hours, which helps to facilitate sleep. It then rises throughout the day, peaking in the late afternoon or early evening to promote wakefulness.
  • c) The cycle in adults is not precisely 27 hours; the intrinsic circadian rhythm is closer to 24 hours but can vary slightly from person to person. This is a common misconception.
  • d) With aging, circadian rhythms tend to advance rather than delay, meaning older adults often feel sleepy earlier in the evening and wake up earlier in the morning, contrary to the statement.
  • e) Adapting to jet lag by advancing the cycle (moving time zones eastward) is often found to be more challenging than delaying it (moving westward), opposite to what is stated. This is because extending our day is usually easier than shortening it.

17-5 (112A6) Regarding sleep, which statement is correct?

a) Dreaming occurs during periods of light non-REM sleep.

b) Deep non-REM sleep decreases towards the morning.

c) REM sleep is characterized by slow eye movements.

d) Infants have less REM sleep than adults.

e) Total sleep time remains constant after adolescence.

 

The correct answer is:

b) Deep non-REM sleep decreases towards the morning.

Explanation:

 

Deep non-REM sleep, also known as slow-wave sleep, predominates in the first third of the night and gradually decreases as the night progresses, especially towards the morning. This phase of sleep is critical for physical restoration and memory consolidation. Other options are incorrect because: a) Dreaming is most often associated with REM sleep, not light non-REM sleep; c) REM sleep is characterized by rapid eye movements, not slow; d) Infants actually have a higher proportion of REM sleep compared to adults; e) Total sleep time typically decreases with age, not remains constant after adolescence.

17-6 (114C13) What is the appropriate guidance for adult patients with insomnia who have difficulty falling asleep?

a) "Wake up at a fixed time and expose yourself to sunlight."

b) "Go to bed as early as possible."

c) "Drink alcoholic beverages before going to bed."

d) "Stay in bed for more than 8 hours a night."

e) "If you can't fall asleep after going to bed, just wait quietly for sleepiness to come."

 

The correct answer is:

a) "Wake up at a fixed time and expose yourself to sunlight."

Explanation:

 

This advice is based on the principle of regulating the circadian rhythm, which is crucial for maintaining healthy sleep patterns. Waking up at the same time every day and exposing oneself to natural sunlight helps to reset the body's internal clock. Sunlight exposure in the morning increases the production of melatonin, a hormone that regulates sleep, in the evening, making it easier to fall asleep at night. The other options could potentially worsen sleep quality or contribute to unhealthy sleep habits. Drinking alcoholic beverages before bed (option c) can disrupt sleep later in the night, and staying in bed for more than 8 hours without sleeping (option d) can lead to increased frustration and anxiety about sleep, potentially worsening insomnia.

17-7 (116C8) Which of the following is correct about REM sleep?

a) It is called slow-wave sleep.

b) Skeletal muscle activity becomes more active.

c) It is less common to have dreams.

d) The activity of the autonomic nervous system is more variable.

e) It accounts for more than half of an adult's sleep in a night.

 

The correct answer is:

d) The activity of the autonomic nervous system is more variable.

Explanation:

 

REM (Rapid Eye Movement) sleep is a phase of sleep characterized by rapid eye movements, increased brain activity, vivid dreams, and atonia (muscle paralysis) to prevent dream enactment. It is during this phase that the autonomic nervous system (ANS) shows more variability, reflecting in fluctuating heart rates and breathing patterns. This variability contrasts with the more stable ANS activity during non-REM (NREM) sleep. REM sleep does not account for more than half of an adult's sleep (e is incorrect); it's actually about 20-25%. REM sleep is not called slow-wave sleep (a is incorrect); that term refers to the deep stages of NREM sleep. Skeletal muscle activity does not become more active; it decreases due to atonia (b is incorrect). It's also during REM sleep that dreams are most vivid and frequently remembered upon waking (c is incorrect).

Clinical

18-1 (109I44) A 15-year-old boy, accompanied by his mother, visited the clinic complaining of abnormal behavior at night. Two weeks ago, around 1 a.m., the mother heard a loud noise from the boy's room. Upon checking, she found him walking around the room with a vacant stare, and a clock was on the floor. When she tried to grab his hand, he suddenly started thrashing violently and she couldn't restrain him, so she went to call the father. When they returned, he was sleeping in bed. The next day, he said he had slept soundly from around 10:30 p.m. until morning and had no recollection of the incident. They visited the clinic after a similar episode occurred last night. Physical examination, blood chemistry, and EEG showed no abnormalities.

The most likely diagnosis is:

a) Night terror

b) Nightmare disorder

c) Sleepwalking

d) REM sleep behavior disorder

e) Circadian rhythm sleep disorder

 

The correct answer is:

c) Sleepwalking

Explanation:

  • The description of the boy's behavior—wandering with a vacant stare, having no memory of the event, and the occurrence during the first half of the night—aligns with sleepwalking (somnambulism). Sleepwalking typically occurs during the deep stages of non-REM (NREM) sleep, early in the night. This condition is characterized by complex behaviors, such as walking around, that occur while the individual is still in a state of sleep. The person often has no memory of these actions.
  • Night terrors (a), which are also known as sleep terrors or pavor nocturnus, are characterized by intense fear and a difficulty to awaken the individual, but without the ambulatory actions typical of sleepwalking.
  • Nightmare disorder (b) involves distressing and vivid dreams that awaken the sleeper, usually during REM sleep, which is not the case here.
  • REM sleep behavior disorder (c) involves acting out vivid, often unpleasant dreams with vocal sounds and sudden, often violent arm and leg movements, typically occurring later in the night during REM sleep phases, which does not match the patient's symptoms.
  • Circadian rhythm sleep disorders (e) are characterized by a chronic or recurrent pattern of sleep-wake rhythm disruption, primarily caused by a misalignment between the body's circadian rhythms and the external environment or by alterations in the internal circadian rhythm system itself, which is not indicated by the symptoms described.

18-2 (111I41) A 65-year-old man, accompanied by his wife, visited the clinic with a complaint of abnormal behavior during sleep. For the past five years, he often experienced vivid nightmares and began speaking clearly in his sleep. Gradually, he started to shout loudly or laugh during sleep, swing his arms around, and sometimes strike his wife. The sleep talking and confused behavior corresponded to the content of his dreams. Which of the following is correct about the most likely condition?

a) There is a high likelihood of transitioning to Alzheimer's dementia.

b) The behavior abnormalities occur in association with the slow-wave sleep phase.

c) Unconscious hostility towards his wife is the cause.

d) Adjustment of the bedroom environment is necessary.

e) Often accompanied by excessive sleepiness.

 

The correct answer is:

d) Adjustment of the bedroom environment is necessary.

Explanation:

 

The scenario described is indicative of REM sleep behavior disorder (RBD), characterized by the enactment of dreams due to a lack of muscle atonia during REM sleep. Patients often exhibit complex behaviors, such as shouting, laughing, or even becoming physically aggressive, which correspond to the vivid, action-packed, or violent dreams they are experiencing. RBD can be associated with neurodegenerative disorders, but the key feature is the loss of normal muscle paralysis during REM sleep, not necessarily a transition to Alzheimer's dementia, making option a incorrect. The behaviors do not specifically occur during slow-wave sleep (option b), nor is there any indication that unconscious hostility towards his wife (option c) or the bedroom environment (option d) is relevant. Excessive sleepiness (option e) is not a typical feature of RBD.

18-3 (112A26) A 67-year-old woman visited the clinic complaining of insomnia. For the past month, she had been experiencing an unpleasant sensation in both legs at night, as if insects were crawling on her. This discomfort intensified when at rest but was alleviated by moving her legs. Although her primary care physician advised her to wait and see if the condition improved, it did not, and the urge to move her legs became so strong that she could not fall asleep, prompting her visit. Muscle tone in her limbs was normal, and there was no muscle weakness. Tendon reflexes were normal, and the Babinski sign was negative. She showed no sensory deficits or cerebellar ataxia, and her walking and daily activities were unaffected. Blood biochemical tests, including serum ferritin, showed no abnormalities.

The appropriate medication for her condition is:

a) Beta-blocker

b) Muscle relaxant

c) Anticholinergic

d) Dopamine receptor agonist

e) Acetylcholinesterase inhibitor

 

The correct answer is:

d) Dopamine receptor agonist

Explanation:

 

The description fits the symptoms of Restless Legs Syndrome (RLS), a neurological disorder characterized by an irresistible urge to move the legs, often accompanied by uncomfortable sensations. These symptoms typically occur in the evening or at night when the person is resting and can significantly impact sleep quality. The cause of RLS is not fully understood, but it is thought to involve dopamine, a neurotransmitter involved in controlling muscle movement. Dopamine receptor agonists, which increase dopamine activity, are effective in treating RLS symptoms by reducing the urge to move the legs and improving sleep. Beta-blockers (a), muscle relaxants (b), anticholinergics (c), and acetylcholinesterase inhibitors (e) are not typically used to treat RLS and do not target its underlying pathophysiology related to dopamine dysfunction.

18-4 (115D31) A 19-year-old male visited the clinic complaining of unbearable daytime sleepiness. Approximately three years ago, he began experiencing sudden bouts of sleepiness during the day, which he initially attributed to fatigue from part-time work or classes. Recently, while being questioned during a university lecture, he was suddenly overcome by sleepiness and fell asleep, prompting concern from a faculty member who recommended a consultation. In addition to sudden sleepiness, he experienced sudden loss of muscle strength and collapsed when laughing hard or being startled during conversations with friends. He also reported feeling a presence or being unable to move during sleep onset, akin to sleep paralysis. Neurological examination and brain MRI showed no abnormalities, and he had no significant medical history.

Which test is useful for the diagnosis?

a) Electromyography (EMG)

b) Auditory brainstem response

c) Holter ECG

d) Polysomnography

e) Dopamine transporter SPECT

 

The correct answer is:

d) Polysomnography

Explanation:

 

This patient's symptoms are characteristic of narcolepsy, a sleep disorder that causes excessive daytime sleepiness, cataplexy (sudden loss of muscle tone triggered by strong emotions), sleep paralysis, and hypnagogic hallucinations (vivid, dream-like experiences while falling asleep). Polysomnography, an overnight sleep study that records brain waves, oxygen levels, heart rate, and breathing, as well as eye and leg movements, is crucial for diagnosing narcolepsy. It can show the presence of REM sleep abnormalities that are indicative of narcolepsy. Electromyography (a), Auditory brainstem response (b), Holter ECG (c), and Dopamine transporter SPECT (e) are not specific tests for diagnosing narcolepsy.

18-5 (116A18) A 23-year-old male, frequently nodding off at work and often falling asleep suddenly during important negotiations or meetings, sought medical advice. Despite getting sufficient sleep at night, he has been experiencing sudden intense drowsiness multiple times during the day for a few years. Recently, he has noticed an increase in episodes where he suddenly loses all muscle strength following laughter or surprise. No abnormalities were found in a neurological examination.

The most likely characteristic observed in this patient is:

a) Experiencing sleep paralysis upon falling asleep.

b) Experiencing delirium at night.

c) Remaining sleepy after sleep attacks.

d) Experiencing panic attacks during sleep.

e) Having a strong urge to move legs during sleep.

 

The correct answer is:

a) Experiencing sleep paralysis upon falling asleep.

Explanation:

 

This symptom, along with the description of episodes of cataplexy (sudden loss of muscle tone triggered by strong emotions such as laughter or surprise), daytime sleep attacks, and no neurological abnormalities or MRI findings, are characteristic features of narcolepsy. Narcolepsy is a chronic sleep disorder that affects the brain's ability to regulate sleep-wake cycles properly. Sleep paralysis is a temporary inability to move or speak while falling asleep or upon waking, which can occur in narcolepsy alongside other symptoms like cataplexy, excessive daytime sleepiness, and sometimes hallucinations. The detailed history matches the narcolepsy symptom complex, especially the combination of excessive daytime sleepiness and cataplexy.

 

Nighttime delirium (b), residual sleepiness after sleep attacks (c), panic attacks during sleep (d), and an increased urge to move legs during sleep (e, a symptom of restless legs syndrome) are not specifically associated with narcolepsy.

18-6 (116D22) A 68-year-old man visited the clinic after being told by his cohabitating family that he talks a lot in his sleep. There is nothing notable in his medical history or family history. According to his wife, who accompanied him, he sometimes shouts loudly at night and kicks his wife next to him by swinging his legs. No abnormalities were found in neurological examinations, blood tests, blood biochemistry tests, or simple head MRI.

The possible disease is:

a) REM sleep behavior disorder

b) Sleep apnea syndrome

c) Restless legs syndrome

d) Narcolepsy

e) Night terrors

 

The correct answer is:

a) REM sleep behavior disorder

Explanation:

 

REM sleep behavior disorder is characterized by the enactment of dreams due to a lack of muscle atonia during REM sleep, leading to potentially violent behaviors and vocalizations. This matches the description of the patient's symptoms, such as shouting and physically acting out during the night. In contrast, the other options listed do not typically involve acting out dreams. Sleep apnea syndrome (b) involves breathing interruptions during sleep, restless legs syndrome (c) involves an irresistible urge to move the legs, narcolepsy (d) is marked by excessive daytime sleepiness and possibly cataplexy, and night terrors (e) involve intense fear and partial awakening from sleep, not the acting out of dreams.

General

19-1 (102A7) Which of the following is true about Attention Deficit Hyperactivity Disorder (ADHD)?

a) It is seen in about 0.5% of children.

b) It is more common in girls.

c) It is accompanied by intellectual disability.

d) Abnormalities in the brain's dopamine neural system are observed.

e) Adrenal cortex steroids are effective.

 

The correct answer is:

d) Abnormalities in the brain's dopamine neural system are observed.

Explanation:

 

Attention Deficit Hyperactivity Disorder (ADHD) is characterized by abnormalities in the brain's dopamine neural system. Dopamine is a neurotransmitter that plays a key role in attention, executive functions, and impulse control, which are areas where individuals with ADHD often have difficulties. This understanding has led to the use of stimulant medications, such as methylphenidate and amphetamines, which increase dopamine levels and activity in the brain, thereby reducing the symptoms of ADHD. Options a, b, c, and e do not accurately describe the epidemiology, gender prevalence, co-morbid conditions, or treatment effectiveness associated with ADHD.

19-2 (104D12) Tics are observed in which of the following?

a) Bipolar disorder

b) Panic disorder

c) Attention Deficit Hyperactivity Disorder (ADHD)

d) Asperger syndrome

e) Tourette syndrome

 

The correct answer is:

e) Tourette syndrome

Explanation:

 

Tourette syndrome is characterized by multiple motor and one or more vocal tics, which are sudden, rapid, recurrent, non-rhythmic motor movements or vocalizations. These symptoms can wax and wane, and they often change in severity and type over time. While ADHD can have associated features like impulsivity, which might manifest as movements or vocalizations, the presence of tics as a primary symptom is specific to Tourette syndrome and not a central feature of ADHD (c), bipolar disorder (a), panic disorder (b), or Asperger syndrome (d).

19-3 (107G16) Among the symptoms seen in Attention Deficit Hyperactivity Disorder (ADHD), which complaint is considered an attention deficit?

a) "I have trouble waiting for my turn."

b) "I get distracted by trivial things easily."

c) "Even when sitting, I end up moving my hands and feet."

d) "I can't write my own name, even though I'm in third grade."

e) "I worry about school items and check them over and over again."

 

The correct answer is:

b) "I get distracted by trivial things easily."

Explanation:

 

This complaint is considered an attention deficit because it directly relates to the difficulty in maintaining focus on tasks or activities, which is a hallmark symptom of ADHD. Attention Deficit Hyperactivity Disorder is characterized by patterns of inattention, hyperactivity, and impulsivity that affect functioning and development. The symptom described in option b reflects the inattention aspect of ADHD, where individuals have a hard time keeping their mind on any one thing and may get bored with a task after only a few minutes, especially if the task is mundane or repetitive.

19-4 (109D3) Which of the following is true about autism?

a) It is characterized by sensory hypersensitivity.

b) Attachment is formed through appropriate parenting.

c) There is intense stranger anxiety and following behavior around the age of 1.

d) Approximately 30% have delayed intellectual development.

e) Once language appears, communication becomes established.

 

The correct answer is:

a) It is characterized by sensory hypersensitivity.

Explanation:

 

People with autism often experience heightened sensitivity to sensory inputs such as sounds, lights, textures, or smells. This can lead to discomfort or even pain in environments that others might find normal, impacting their daily lives and behaviors. This characteristic helps in understanding some of the behaviors and needs of individuals with autism, emphasizing the importance of creating supportive and accommodating environments for them. The other options do not accurately describe characteristics of autism: proper parenting does not necessarily affect the formation of attachment in the context of autism (b), intense stranger anxiety and following behavior are not specific or definitive indicators of autism (c), not all individuals with autism have delayed intellectual development, and it varies greatly among individuals (d), and the emergence of language does not guarantee the establishment of effective communication (e).

19-5 (111I14) Which of the following is true about Attention Deficit Hyperactivity Disorder (ADHD)?

a) It is accompanied by attachment disorders.

b) Language development is delayed.

c) It is more common in girls than in boys.

d) The primary cause is the parenting attitude.

e) Hyperactivity worsens in new situations.

 

The correct answer is:

e) "Hyperactivity worsens in new situations."

Explanation:

 

ADHD is characterized by patterns of inattention, hyperactivity, and impulsivity that are more severe, frequent, or interfere more with daily functioning than typically observed in individuals at a comparable level of development. While a, b, c, and d are not universally true statements about ADHD, e is accurate because individuals with ADHD may exhibit increased hyperactivity and impulsivity in new or stimulating environments. This can be due to the novelty and abundance of stimuli in such settings, which may overwhelm their ability to regulate attention and activity levels. This characteristic is part of the diagnostic criteria for ADHD, as it reflects the difficulty individuals with this condition have in modulating their behavior according to the demands of different situations.

19-6 (114A8) Regarding selective mutism, which of the following is correct?

a) It is common in adolescence.

b) Hearing is normal.

c) It is caused by a language comprehension disorder.

d) EEG is useful for diagnosis.

e) Treatment with vocal exercises is effective.

 

The correct answer is:

b) Hearing is normal.

Explanation:

 

Selective mutism is a complex childhood anxiety disorder characterized by a child's inability to speak in certain social situations, such as at school or in the community, despite speaking in other situations, like at home with family members. The disorder is not due to a lack of knowledge of, or comfort with, the spoken language required in the social situation, nor is it due to a communication disorder (e.g., childhood-onset fluency disorder). The key characteristic of selective mutism is that the child has a normal ability to understand and use language, but fails to speak in specific situations due to anxiety, fear, or shyness. Hearing is typically normal in these individuals, as selective mutism is a psychological rather than physiological condition. Other options are incorrect as they do not accurately describe features or effective treatments related to selective mutism.

19-7 (115F4) In adults, which symptom is more frequently observed in Attention Deficit Hyperactivity Disorder (ADHD) compared to Autism Spectrum Disorder?

a) Lack of eye contact.

b) Not understanding jokes.

c) Frequent careless mistakes.

d) Many off-target responses.

e) Obsession with being symmetrical.

 

The correct answer is:

c) Frequent careless mistakes.

Explanation:

 

ADHD is characterized by difficulties in maintaining attention, impulsivity, and in some cases, hyperactivity. Individuals with ADHD often make careless mistakes due to difficulty sustaining attention on tasks or activities. This symptom is more prevalent in ADHD than in Autism Spectrum Disorder, where challenges primarily involve social communication and fixed interests or repetitive behaviors.

19-8 (116A11) Which of the following is not a characteristic of Autism Spectrum Disorder?

a) More common in boys.

b) Sensory hypersensitivity.

c) Difficulty with social relationships.

d) Symptoms are observable before school age.

e) Weak obsession with objects.

 

The correct answer is:

e) Weak obsession with objects.

Explanation:

 

Autism Spectrum Disorder (ASD) is characterized by strong and persistent interests or fixations on specific topics or routines, which is contrary to having a "weak" obsession. Individuals with ASD often exhibit intense focus and deep knowledge on subjects of interest, and they may insist on following routines or rituals with rigidity. This can sometimes be a coping mechanism to manage the sensory and social complexities of their environment. In contrast, the other options listed (a-d) are common characteristics associated with ASD, including being more common in boys, experiencing sensory hypersensitivity, having difficulty with social relationships, and the early emergence of symptoms before school age.

Clinical

20-1 (101G6) A 12-year-old boy stopped attending middle school claiming he was being bullied by friends, leading his mother to bring him in for a consultation. Despite normal physical and language development and consistently high academic performance, he has struggled to make friends since childhood, often preferring solitary activities. He avoids making eye contact when speaking with others, shows limited emotional expression, and does not share joy with others. He finds it difficult to read the atmosphere and act or speak accordingly, leading his schoolmates to perceive him as selfish. His interests are limited; he collects model cars from around the world and has memorized nearly all their features. There are no experiences of hallucinations or delusions.

Which of the following is observed in this disorder?

a) Nocturnal enuresis

b) Attention deficit

c) Vocal tic

d) Repetitive behaviors

e) Development into conduct disorder

 

The correct answer is:

d) Repetitive behaviors

Explanation:

 

This boy's behavior aligns with characteristics often seen in autism spectrum disorder (ASD). Individuals with ASD may have normal to high intelligence and specific academic strengths but struggle with social interactions, emotional expression, and nonverbal communication, such as making eye contact. A hallmark feature of ASD is engaging in repetitive behaviors and having intensely focused interests, as demonstrated by the boy's extensive knowledge and collection of model cars. Unlike the other options listed, repetitive behaviors and restricted interests are core symptoms of ASD. Nocturnal enuresis (a), attention deficit (b), vocal tic (c), and development into conduct disorder (e) are not specifically indicative of ASD and do not directly relate to the symptoms described in this scenario.

20-2, 3, 4 (102G64, 102G65, 102G66) A 21-year-old male presented to the clinic concerned about his future, including job prospects.

Past Medical History: As he was preparing to graduate from university and undergoing job interviews at various companies, he experienced fear in front of interviewers and was unable to speak.

Birth and Developmental History: Born at 38 weeks by natural delivery. Although he was clumsy with his hands, there was no delay in language development. During his early childhood, he seldom made eye contact with others and often played alone. In elementary school, he had a keen interest in professional baseball players' numbers and careers, earning him the nickname "Baseball Encyclopedia." Throughout elementary, middle, and high school, he struggled with sports and often faced bullying and teasing. He then entered the engineering department of his desired university as an active student. He has few friends and no experience in romantic relationships.

Past Health History: Nothing noteworthy.

Current Condition: Conscious and alert. Height 172 cm, weight 55 kg. Body temperature 36.1°C, pulse 76 beats per minute, regular. Blood pressure 120/78 mmHg. Appeared anxious in the consulting room, looking down and avoiding eye contact.

1. Which two conditions are identified in this patient?

a) Intellectual disability

b) Sleep disorder

c) Memory disorder

d) Social interaction disorder

e) Restricted interests and behaviors

2. Which test is most useful for diagnosis?

a) Psychological assessment

b) EEG (Electroencephalogram)

c) Cerebrospinal fluid analysis

d) Brain SPECT e) Simple MRI of the head

3. What is the most likely diagnosis?

a) Conduct disorder

b) Childhood autism

c) Schizophrenia

d) Asperger syndrome

e) Attention Deficit Hyperactivity Disorder (ADHD)

 

1. The correct answers are:

d) Social interaction disorder

e) Restricted interests and behaviors

Explanation:

 

Given the patient's symptoms and history, the most relevant information to consider includes difficulty in social interactions, restricted interests (e.g., profound knowledge of baseball statistics), lack of eye contact, and difficulties during social situations such as job interviews. There's no mention of significant issues with intellectual functioning, memory problems, or specific sleep disorders. Therefore, we can deduce:

  • d) Social interaction disorder: The patient's difficulty with eye contact, preference for solitude, and challenges in social situations such as job interviews suggest issues with social interactions.
  • e) Restricted interests and behaviors: His intense interest in a specific topic (baseball statistics) to the point of being nicknamed and the lack of varied, spontaneous play or social engagement suggests restricted interests and repetitive behaviors.
2. The correct answer is:
a) Psychological assessment
Explanation:
  • a) Psychological assessment: A comprehensive psychological evaluation is crucial for understanding the patient's social functioning, interests, behaviors, and possible sensory sensitivities. It can help diagnose conditions on the autism spectrum, including Asperger syndrome, by evaluating social communication skills and the presence of restricted, repetitive patterns of behavior, interests, or activities.
3. The correct answer is:
d) Asperger syndrome
Explanation:
  • d) Asperger syndrome: This diagnosis aligns with the patient's significant social interaction difficulties, restricted interests, and the absence of language or cognitive development delay. Asperger syndrome is a condition on the autism spectrum characterized by these features. While the DSM-5 has integrated Asperger syndrome under the broader category of Autism Spectrum Disorder (ASD), it's still commonly referred to individually, especially in contexts predating the DSM-5.

The selected answers highlight the patient's challenges with social interactions and specialized interests, pointing towards a diagnosis on the autism spectrum, with Asperger syndrome being a fitting categorization based on the provided history and symptoms.

20-5 (103A23) A 4-year-old boy was brought to the clinic because he does not show interest in other children at daycare. He is 102 cm tall and weighs 15.6 kg. He did not follow his mother around in infancy and currently prefers to play alone. He started speaking late and cannot speak in two-word sentences. He has an unusual interest in ventilation fans.

Which of the following are observed in this disorder? Choose two.

a) Repeating the same action.

b) Echolalia.

c) Being shy with strangers.

d) Making eye contact.

e) Enjoying pretend play.

 

The correct answers are:

a) Repeating the same action.

b) Echolalia.

Explanation:

 

For the 4-year-old boy described, the most likely diagnosis is an autism spectrum disorder (ASD). ASD is characterized by challenges with social skills, repetitive behaviors, speech and nonverbal communication, as well as by unique strengths and differences. The key features described in the scenario, such as not showing interest in other children, preferring solitary play, delayed speech development, and having an unusual interest (e.g., in ventilation fans), are typical of ASD. The answers to the question are:

  • a) Repeating the same action. - Correct. Children with ASD often engage in repetitive behaviors or have restricted interests, such as focusing intently on one item to the exclusion of others.
  • b) Echolalia. - Correct. Echolalia, the exact repetition of phrases without necessarily understanding their meaning, is a common speech pattern in children with ASD.
  • c) Being shy with strangers. - While children with ASD might appear shy or withdrawn, shyness with strangers is not a distinctive characteristic of ASD. The core issue is more about difficulty with social interactions and communication.
  • d) Making eye contact. - Incorrect. Many children with ASD find it difficult to maintain eye contact during social interactions, which is part of their challenges with nonverbal communication.
  • e) Enjoying pretend play. - Incorrect. Children with ASD typically have difficulty engaging in pretend play, which involves imagination and social interaction. They might not pretend to be characters from stories or engage in games that involve "pretending."

20-6 (107I44) A 10-year-old boy was brought to the clinic by his parents, complaining of uttering obscene words. Since about the age of 5, he has started exhibiting sudden, rhythmic, repetitive movements such as blinking, shrugging, head shaking, and clearing his throat, with periods of remission and exacerbation. The complaint that brought them in has recently started. There is no significant past medical history, and neurological examination shows no abnormalities.

Regarding this child, which of the following is correct?

a) Parental over-involvement is the main cause.

b) Abnormal movements nearly disappear during sleep.

c) There is little variation in symptoms depending on the situation.

d) Symptoms often persist without improvement until adulthood.

e) It is impossible to stop the symptoms, even for a very short time.

 

The correct answer is:

b) Abnormal movements nearly disappear during sleep.

Explanation:

 

This scenario is indicative of Tourette syndrome, a neurological disorder characterized by repetitive, stereotyped, involuntary movements and vocalizations called tics. The key aspects that point towards this diagnosis include the onset in childhood, the presence of both motor and vocal tics, and the fluctuating nature of the symptoms with periods of exacerbation and remission.

  • Option b is correct because it is a well-documented feature of Tourette syndrome and other tic disorders that the tics generally subside during sleep. This characteristic can help distinguish Tourette syndrome from other conditions that might cause similar symptoms.

The other options are incorrect for the following reasons:

  • a) Tourette syndrome is not caused by parental behavior or family dynamics. It is a neurodevelopmental disorder with a genetic component.
  • c) Symptoms of Tourette syndrome can indeed vary significantly depending on environmental and situational factors. Stress, excitement, or being focused on activities can influence the frequency and intensity of tics.
  • d) While it's true that for some individuals with Tourette syndrome, symptoms persist into adulthood, many people experience a significant reduction in tics by late adolescence or early adulthood.
  • e) Individuals with Tourette syndrome can sometimes suppress their tics for a short period, especially when they are aware that attention is focused on them or in certain social contexts, but this suppression often leads to a rebound increase in tics later.

20-7 (108D20) An 8-year-old boy was brought to the clinic by his mother because he did not want to go to school. Although his grades are average, he struggles with writing, especially copying kanji characters accurately. This has led to frequent scolding from teachers and parents, making him reluctant to go to school. He has no problems with his peer relationships and performs normally in sports. Although he is somewhat clumsy, no other abnormalities were found in a neurological examination.

Which of the following about this condition is incorrect?

a) It is more common in boys.

b) Intelligence is often within the normal range.

c) There is a high comorbidity with attention deficit/hyperactivity disorder.

d) The fundamental symptoms often persist into adulthood.

e) Writing often improves when the individual is seriously engaged in the task.

 

The incorrect statement about this condition is:

e) Writing often improves when the individual is seriously engaged in the task.

Explanation:

 

This statement is incorrect because the boy's difficulty with writing, especially copying kanji accurately, suggests a specific learning disorder with impairment in writing (dysgraphia). In cases of dysgraphia, the issue isn't related to how seriously the individual is engaged in the task. Dysgraphia is a neurological disorder characterized by a significant difficulty with spelling, handwriting, and organizing ideas on paper, and it can't simply be improved by trying harder or being more seriously engaged in the task. Dysgraphia requires specific interventions and strategies to help the individual improve their writing skills, which often include accommodations, such as the use of typing devices, specific teaching strategies, and therapy.

20-8 (109A54) An 8-year-old boy was brought to the clinic by his mother due to his restlessness. Since his toddler years, he has shown signs of restlessness and has had difficulty following turns and rules during play. He sometimes leaves his seat during class and is reluctant to do his homework at home, rarely completing it. His academic performance is average, and no abnormalities were found in his physical examination.

The most appropriate initial response is:

a) Introduce pharmacotherapy.

b) Strictly reprimand for problematic behavior.

c) Seat him at the back of the class where he can see the entire classroom.

d) Consider the duration he can concentrate for and assign tasks accordingly.

e) Explain to the mother that it will improve as he becomes an adult.

 

The most appropriate initial response is:

d) Consider the duration he can concentrate for and assign tasks accordingly.

Explanation:

 

This approach acknowledges the child's current capacity for attention and attempts to work within those limits rather than against them. It's a strategy that can help manage symptoms of ADHD (Attention Deficit Hyperactivity Disorder), for which this child might be showing signs. Introducing pharmacotherapy (a) might be considered at some point but typically follows a thorough evaluation and often after trying behavioral strategies. Strict reprimands (b) could exacerbate the problem by increasing stress and anxiety. Seating him at the back of the class (c) doesn't directly address his restlessness or difficulty concentrating. Telling the mother that it will improve as he becomes an adult (e) overlooks the immediate need for strategies to help the child manage his symptoms.

20-9 (110A42) A 3-year-old boy was brought to the clinic by his parents due to not speaking. There were no issues with his motor development, but he was not speaking. In the examination room, he did not make eye contact, moved around, rocked his body back and forth, and often spun around. When he found building blocks and started playing with them, he focused intensely on lining them up in a row and did not respond to his parents calling him. There was no delay in motor development, and no signs of hearing impairment were observed.

The correct statement about this disorder is:

a) Picky eating is rare.

b) Understanding of language is good.

c) The parents' parenting attitude is the cause.

d) Symptoms fully develop in adolescence.

e) There is an impairment in communicating one's own intentions.

 

For the 3-year-old boy described, the correct answer is:

e) There is an impairment in communicating one's own intentions.

Explanation:

 

This child displays behaviors indicative of Autism Spectrum Disorder (ASD), which includes difficulty in establishing eye contact, repetitive movements (such as rocking and spinning), and not responding to his name. Children with ASD often have significant difficulties with both verbal and non-verbal communication. They might not speak at all or have delayed language development. When they do communicate, they might have trouble using their language skills to convey their needs, desires, or thoughts effectively. The focus on lining up toys and ignoring social cues like a parent calling their name is characteristic of ASD. The impairment in communicating one's intentions is a hallmark of the disorder, distinguishing it from conditions where speech might be delayed but the understanding and desire to communicate are intact.

 

Options a) through d) do not accurately describe the typical features of ASD in relation to the description given:

  • Picky eating (a) can be common in children with ASD due to sensory sensitivities.
  • Understanding of language (b) can be impaired in ASD, contrary to the statement provided.
  • The condition is not caused by parenting attitudes (c), but is a neurodevelopmental disorder.
  • While ASD symptoms can evolve, they typically start in early childhood (d), not adolescence.

20-10 (111D43) A 5-year-old boy was brought to the clinic by his parents because he doesn't play with other children at kindergarten. There are no issues with his motor or language development, but it's hard for him to make eye contact, and he responds poorly to being called. He is deeply fascinated by a train encyclopedia and remembers many car names. He often plays alone at kindergarten. In the consulting room, he can converse but cannot sit still and talks unilaterally about things he is interested in. No abnormalities were found in the physical examination.

Which of the following is correct about this child?

a) Intellectual disability coexists.

b) Transitions to schizophrenia.

c) Self-care independence is good.

d) Social development is good.

e) Coexistence with tic disorders is rare.

 

The correct answer is:

c) Self-care independence is good. 

Explanation:

 

This scenario describes a child who likely has autism spectrum disorder (ASD). ASD is characterized by challenges in social interaction, restricted interests, and repetitive behaviors. Despite these challenges, many individuals with ASD can achieve a good level of self-care independence, especially when their intellectual ability is within the normal range, as suggested in this scenario by the absence of developmental delays in motor or language abilities. This independence in self-care is an important marker of functionality and can significantly contribute to their quality of life. The emphasis on the child's ability to engage deeply with subjects of interest (like the train encyclopedia) and to remember detailed information, alongside challenges in social interaction and non-standard communication patterns, supports the diagnosis of ASD without necessarily indicating intellectual disability or predisposition to other conditions such as schizophrenia or tic disorders.

20-11 (115A54) A 6-year-old boy was brought to the clinic by his mother, complaining of shaking his head from side to side. This action occurred for about 5 minutes while watching TV after lunch, prompting the visit. He had been noticed to blink rapidly 7 months ago, which subsided after about 2 months. Four months ago, the blinking resumed, accompanied by sudden, non-rhythmic grimacing and shaking his head from side to side. These symptoms do not appear during sleep and occur more frequently when relaxed. He is able to suppress them for short periods. Neurological examination shows no abnormalities.

Which of the following is the most likely diagnosis?

a) Tremor

b) Tic

c) Ballismus

d) Chorea

e) Athetosis

 

The most likely diagnosis is:

b) Tic

Explanation:

 

This diagnosis is supported by the presentation of sudden, repetitive, non-rhythmic motor movements (e.g., blinking, head shaking) and vocalizations that the child can voluntarily suppress for short periods. Tics often increase with stress or relaxation and decrease during sleep or focused activities. The ability of the child to suppress these movements temporarily and the absence of symptoms during sleep are characteristic of tic disorders. Other options like tremor (a), ballismus (c), chorea (d), and athetosis (e) involve movements that are typically continuous, more widespread, and not suppressible in the manner described.

General

Clinical

22-1 (115D28) A 25-year-old woman has been engaging in self-harm behaviors like cutting herself due to issues in her romantic and workplace relationships, leading to a visit to the psychiatric department accompanied by her mother. She claims the visit is due to irritability and insomnia. It was discovered she had sexual relationships with several male colleagues at her recent workplace, which made her feel uncomfortable and led to her resignation. She has exhibited behaviors such as repeatedly making late-night calls to close friends and a former boss, and when cautioned, she reacts aggressively by yelling and insulting. When others become fed up and distance themselves, she hints at suicide on social media and has even called an ambulance for herself. On the other hand, when she is in a good mood, she gives presents to those she likes or sends them numerous affectionate emails, indicating severe emotional fluctuations.

The characteristic expected to be seen in this patient is:

a) Repeatedly lying.

b) Obsessing over sixth sense or superstitions.

c) Holding chronic feelings of emptiness.

d) Being unable to finish tasks due to perfectionism.

e) Seeking to be the center of attention.

 

The correct answer is:

c) Holding chronic feelings of emptiness.

Explanation:

 

This woman's behavior is indicative of Borderline Personality Disorder (BPD), a mental health disorder characterized by pervasive instability in moods, behavior, self-image, and functioning. These individuals often experience intense episodes of anger, depression, and anxiety that can last from a few hours to days. One of the hallmark features of BPD is a chronic feeling of emptiness. Patients with BPD may engage in self-harm or suicidal behavior, have intense and unstable relationships, and show significant mood swings. The behaviors described, including the pattern of intense and unstable relationships, self-harming behaviors as a response to distress, and significant mood swings, align with the diagnostic criteria for BPD, particularly the chronic feeling of emptiness that many individuals with BPD experience.

General

23-1 (102B21) Which of the following is true about rehabilitation for individuals with mental disorders?

a) The effects are temporary.

b) Employment support is not included.

c) Occupational therapy is not conducted during hospitalization.

d) It is often combined with pharmacotherapy.

e) Daycare is not provided for outpatients.

 

The correct statement about rehabilitation for individuals with mental disorders is:

d) It is often combined with pharmacotherapy.

Explanation:

 

Rehabilitation for individuals with mental health issues encompasses a wide range of services and interventions, including occupational therapy, psychotherapy, support for employment, and various forms of social rehabilitation. These services are designed to improve functioning, reduce symptoms, and enhance the quality of life. Combining these interventions with pharmacotherapy is common and often necessary to address both the symptoms and the underlying causes of the disorder effectively. Pharmacotherapy can manage symptoms, making the individual more receptive to rehabilitation efforts, which in turn, can help the individual develop skills and strategies for better managing their condition and leading a more fulfilling life.

23-2 (111B12) Which of the following is primary prevention for mental disorders?

a) Stress checks at the workplace

b) Employment support for schizophrenia

c) Cognitive therapy for alcohol dependence

d) Continuation of pharmacotherapy after remission of depressive disorders

e) Administration of anxiolytics for situational anxiety in anxiety disorders

 

The correct answer is:

a) Stress checks at the workplace

Explanation:

  • Primary prevention in mental health aims to prevent the occurrence of mental disorders before they start. Stress checks at the workplace can help identify individuals at risk of developing stress-related disorders early on, allowing for interventions that could prevent the progression to more serious mental health conditions. This approach aligns with the concept of primary prevention, which focuses on reducing risk factors and enhancing protective factors to prevent the onset of illness.
  • The other options, such as employment support for schizophrenia (b), cognitive therapy for alcohol dependence (c), continuation of pharmacotherapy after remission of depressive disorders (d), and administration of anxiolytics for situational anxiety in anxiety disorders (e), are more aligned with secondary or tertiary prevention strategies aimed at reducing the impact of existing conditions or preventing relapse.

23-3 (115C5) When a cancer patient expresses a desire to die, the correct initial response is:

a) Prescribe antidepressants.

b) Refuse future medical treatment.

c) Explain that suicide causes trouble for those around them.

d) Recommend consulting with psychiatry.

e) Ask what is making them feel that way.

 

The correct answer is:

e) Ask what is making them feel that way.

Explanation:

 

When a cancer patient expresses a desire to die, it is crucial to provide a supportive and empathetic response. Asking the patient about their feelings and the reasons behind such thoughts allows for an open dialogue. This approach shows empathy and understanding, acknowledging the patient's distress without immediately jumping to solutions or judgments. It helps in understanding the patient's emotional state and underlying issues, which is essential for providing appropriate support and interventions. This response respects the patient's feelings and signals a willingness to listen and help, which is a vital first step in addressing their emotional and psychological needs.

Clinical

24-1 (107G40) A 48-year-old man was brought to the clinic by police officers. He had been observed living in a park for the past two weeks. Starting two days ago, he began to exhibit incomprehensible behavior, though he had not shown any signs of harming others. The park's manager reported him, resulting in the clinic visit. Upon examination, he was found to be in a state of hallucinations and delusions, with insufficient communication, making hospitalization necessary. His temperature was 36.8°C, pulse was 88 bpm, and blood pressure was 136/88 mmHg. Despite explaining the need for hospitalization, consent could not be obtained. A medical card from a distant healthcare facility was found among his belongings, identifying him. It seems he has a spouse, but contact could not be made.

Which type of admission under the Mental Health and Welfare for Persons with Mental Disorders Act is appropriate?

a) Emergency Admission

b) Involuntary Admission

c) Voluntary Admission

d) Medical Protection Admission

e) Urgent Protective Admission

 

The correct answer is:

a) Emergency Admission

Explanation:

 

The patient's situation was considered an immediate risk to his health or safety, necessitating urgent intervention even without the ability to obtain consent from the patient or his family. This type of admission is used in situations where there is an immediate need to address the patient's condition to prevent significant harm or deterioration of their mental state. It's an essential mechanism for ensuring that individuals who are unable to understand or participate in their care decisions due to their mental state receive the necessary treatment promptly.

24-2 (109G49) A 20-year-old male, accompanied by his family, visited the clinic primarily concerned about not attending university. He had started university two years ago but stopped attending after six months and has since led a nocturnal lifestyle. While he participates in gatherings related to his hobbies about once a month and occasionally goes shopping, he mostly stays in his room, spending his days playing games on the internet. Though irregular, he maintains his meals and bathing. He mentioned coming to the clinic "because he had to" but was polite, responded accurately to questions, and exhibited natural facial expressions. No physical abnormalities were found.

The most appropriate initial response would be:

a) Encourage him to go to university.

b) Prohibit internet use.

c) Instruct him to keep a sleep diary.

d) Advise increasing outings related to hobbies.

e) Discuss if there are any concerns about his current lifestyle.

 

The correct answer is :

e) Discuss if there are any concerns about his current lifestyle.

Explanation:

 

In a case like this, where a young adult is exhibiting withdrawal from social and educational activities without any apparent physical health issues, the priority is understanding the underlying issues rather than enforcing immediate behavioral changes. The mention of maintaining basic hygiene, participating in hobbies, and the ability to engage appropriately during the consultation suggests there might not be a severe psychiatric disorder at play. However, his avoidance of university and preference for a nocturnal lifestyle indicate potential underlying issues, such as social anxiety, depression, or adjustment disorders, that need to be addressed.

 

Engaging the patient in a conversation about his current lifestyle allows the healthcare provider to assess his mental state, understand his perspective, and identify specific challenges he's facing. This approach is more likely to build rapport and trust, making it easier to explore sensitive topics and provide tailored advice or interventions. Encouraging him to talk about his life, feelings, and potential stressors can reveal important information for formulating an appropriate and effective treatment plan, whether that involves therapy, lifestyle adjustments, or exploring interests that could motivate him to participate more actively in life outside the internet and gaming.

24-3 (110G54) A 12-year-old boy, accompanied by his parents, visited the clinic with the main complaint of being unable to go to school. He had entered junior high school in April, but since the beginning of May, he started experiencing headaches and abdominal pain in the mornings and began to miss school, becoming completely unable to attend by July. Before going to bed at night, he says, "I will go to school tomorrow" and prepares, but when his parents encourage him to go to school and accompany him, he can go up to the school gate but cannot enter the school. He does not go out but spends time at home with his hobbies. He visited a nearby clinic and underwent physical examinations but no abnormalities were found, and he visited this clinic upon referral. During the visit, he was polite and responded appropriately. He says, "I think I have to go to school, but I can't."

The most appropriate current response would be:

a) Suggest transferring schools.

b) Recommend hospitalization.

c) Prescribe anti-anxiety medication.

d) Introduce hypnotherapy.

e) Instruct the parents not to force him to attend school.

 

The most appropriate response at this point is:

e) Instruct the parents not to force him to attend school.

Explanation:

 

Forcing a child who is unable to attend school due to psychological distress can exacerbate their anxiety and reluctance to attend school. It's essential to address the underlying issues causing the school avoidance, such as anxiety or stress, in a supportive and understanding manner. The goal should be to gradually reintegrate the child into the school environment without causing further psychological harm. This approach encourages a positive association with school and learning, promoting a healthier reintegration into the school system. It's also important for the parents to work closely with educators and mental health professionals to develop a comprehensive plan tailored to the child's specific needs.

24-4 (111G54) A 15-year-old girl visited the clinic with her mother, mainly concerned about her refusal to attend school. Since the upper grades of elementary school, she has been teased by her peers for looking like a boy, a situation that continued after entering middle school, leading to her frequently missing school. At the time of the visit, she responded accurately to questions, showed no issues in her thought process, and expressed her emotions naturally. When asked about the reason for not attending school, she mentioned, "I get bullied by friends, but I don't really mind it. It was okay in elementary school because we wore plain clothes, but in middle school, we have to wear uniforms, and I don't want to wear a skirt. I also hate menstruating; it was really tough when it started. I don't want anyone to see my chest getting bigger, so I don't want to go to school. I thought nobody would understand if I talked about this, so I haven't told anyone." No apparent abnormalities in physical development were noted, including neurologic findings.

The most likely choice is:

a) Sexual preference disorder

b) Delusional disorder

c) Gender identity disorder

d) Ego-dystonic sexual orientation

e) Personality disorder

 

The most likely diagnosis for this 15-year-old girl is:

c) Gender identity disorder

Explanation:

 

Her statements about discomfort with wearing a skirt, dissatisfaction with menstruating, and distress over her developing breasts suggest a significant incongruence between her experienced gender and the gender assigned at birth. This condition, where a person's gender identity does not align with their biological sex, leading to distress, is characteristic of gender identity disorder (now more commonly referred to as gender dysphoria in many clinical settings). Her avoidance of school due to these feelings and her belief that others may not understand her situation further supports this diagnosis.

24-5 (114F41) A 25-year-old man, accompanied by his brother, visited the clinic with complaints of hearing voices. He started hearing distressing voices since yesterday, criticizing his thoughts and commanding his actions even though no one was around, causing him significant distress. Due to these symptoms, he sought treatment at a psychiatric hospital where he was previously diagnosed with schizophrenia three years ago and has been under outpatient care. The patient expresses a desire for hospitalization to eliminate the voices he hears.

The most appropriate type of hospital admission is:

a) Emergency admission

b) Voluntary admission

c) Involuntary admission

d) Medical protection admission

e) Emergency intervention admission

 

The most appropriate type of hospital admission for this patient is:

c) Involuntary admission

Explanation:

 

Given the patient is already under outpatient care for schizophrenia and is experiencing distressing symptoms, he expresses a desire for hospitalization to receive treatment aimed at alleviating these symptoms. "Involuntary admission" might not directly translate to the terms used in different jurisdictions, but in this context, it refers to admission initiated by the patient's request or consent, acknowledging his current need for more intensive care. This choice aligns with the patient's expressed willingness for treatment to manage his symptoms effectively, thus suggesting that "any voluntary or patient-consented admission" would be the most fitting option based on the given choices and information.

24-6 (115F55) A 22-year-old male, who had been isolating himself in his room at home and frequently uttering unfounded claims such as "Stop badmouthing me" and "People passing by are peeking in through the window," visited the clinic with his parents. The psychiatric evaluation, conducted by a mental health designated physician, deemed treatment necessary but noted the patient's lack of willingness to undergo treatment. As a result, he was admitted for medical protection.

The restrictions that can be applied include:

a) Meetings with a lawyer

b) Letters to parents

c) Phone calls to friends

d) Overnight stays outside the home as per family wishes

e) Requests for discharge to the Mental Health Review Board

 

The correct answers are:

c) Phone calls to friends

d) Overnight stays outside the home as per family wishes

Explanation:

  

In the context of medical protective hospitalization, restrictions can be placed on a patient's communication and movements to ensure their safety and the effectiveness of treatment. However, these restrictions must balance the need for treatment with the rights of the patient.

 

While a patient under medical protective hospitalization may have certain freedoms limited, such as the ability to make unsupervised phone calls (c) or leave the hospital for overnight stays without explicit permission (d), essential rights like communicating with a lawyer (a) or sending letters (b) are typically preserved to protect the patient's legal rights and ensure their access to external advocacy. Moreover, the patient retains the right to request discharge through the Mental Health Review Board (e), ensuring a mechanism for review and oversight of the hospitalization decision. These measures are designed to protect the patient's well-being while also safeguarding their rights and dignity during treatment.

24-7 (116C49) A 75-year-old woman, accompanied by her distressed husband, visited the clinic due to her nocturnal wandering. They live together, with her husband being 78 years old. She was diagnosed with Alzheimer's disease at the age of 60, which has progressed over time. Recently, in the past three months, she often left the house alone, wandered far from home, and was frequently found and protected by the police. It was decided for her to be admitted to a psychiatric ward for thorough examination and treatment of her wandering and insomnia. She barely speaks and her intentions are unclear. No cognitive decline is observed in her husband.

The most appropriate form of hospitalization is:

a) Emergency measures hospitalization

b) Dispositional hospitalization

c) Emergency hospitalization

d) Medical protection hospitalization

e) Voluntary hospitalization

 

The correct answer is:

d) Medical protection hospitalization

Explanation:

 

In the context of the given scenario, where a patient with advanced Alzheimer's disease is wandering at night and unable to express her intentions, medical protection hospitalization is most appropriate. This type of hospitalization is used when a patient is unable to make decisions due to their mental condition and requires protection and treatment in a hospital setting. Since the patient has a significant cognitive impairment that puts her safety at risk, and she is not capable of consenting to treatment or understanding her condition, medical protection hospitalization ensures she receives the necessary care and protection. This option is chosen over others because it allows for the patient to be admitted without her direct consent, given her inability to express or make decisions, and aims to safeguard her well-being while providing necessary medical care.

General

25-1 (101B89) Which is correct?

a) The Mini-Mental State Examination (MMSE) is used for the evaluation of developmental disabilities.

b) The Hamilton Depression Rating Scale is a self-administered rating scale.

c) The Rorschach test is a personality test based on the questionnaire method.

d) The Wechsler Adult Intelligence Scale (WAIS) is used for the evaluation of depression.

e) The Brief Psychiatric Rating Scale (BPRS) is used for the evaluation of psychiatric symptoms.

 

The correct answer is:

e) The Brief Psychiatric Rating Scale (BPRS) is used for the evaluation of psychiatric symptoms.

Explanation:

 

The BPRS is a widely used instrument for assessing the severity of various psychiatric symptoms such as anxiety, depression, hallucinations, and thought disorders. It's not self-administered but rather completed by a clinician based on an interview with the patient. This tool helps in monitoring and measuring changes in psychiatric symptoms over time, making it a valuable asset in both clinical and research settings.

25-2 (103B4) Which of the following is used for the assessment of frontal lobe function?

a) Rorschach Test

b) Standard Higher Visual Perception Test

c) Minnesota Multiphasic Personality Inventory (MMPI)

d) Mini-Mental State Examination (MMSE)

e) Wisconsin Card Sorting Test (WCST)

 

The correct answer is:

e) Wisconsin Card Sorting Test (WCST)

Explanation:

 

The WCST is widely used to assess frontal lobe functions because it evaluates an individual's ability to form abstract concepts and to shift cognitive strategies in response to changing environmental contingencies. This test measures executive functions, such as strategic planning, organized searching, utilizing environmental feedback to shift cognitive sets, directing behavior toward achieving a goal, and modulating impulsive responding. These functions are closely associated with the frontal lobes of the brain.

25-3 (107B25) Which of the following tests is appropriate for a 3-year-old girl suspected of delayed development in language and cognition?

a) Rorschach Test

b) Tsunoda-Inage Developmental Test

c) Standard Language Test of Aphasia (SLTA)

d) Wechsler Intelligence Scale for Children (WISC-III)

e) Mini-Mental State Examination (MMSE)

 

The correct answer is:

b) Tsunoda-Inage Developmental Test

Explanation:

  • The Tsunoda-Inage Developmental Test is specifically designed for assessing the development of children, including language and cognition, from infancy to early childhood. It evaluates various aspects of development, such as gross and fine motor skills, language comprehension and expression, social interaction, and cognitive abilities, making it suitable for assessing a child's overall development at this age.
  • The Rorschach Test (a) is a projective psychological test primarily used with adolescents and adults to examine personality characteristics and emotional functioning.
  • The Standard Language Test of Aphasia (SLTA) (c) is designed for assessing language impairments in individuals who have suffered brain damage due to stroke or injury, typically used with adults.
  • The Wechsler Intelligence Scale for Children (WISC-III) (d) is a standardized intelligence test designed for children aged 6 to 16, not appropriate for assessing a 3-year-old's cognitive development.
  • The Mini-Mental State Examination (MMSE) (e) is a brief screening tool for assessing cognitive impairment in adults, particularly used in geriatric populations, and is not suitable for assessing a 3-year-old's cognitive development.

25-4 (109G37) Questionnaire-based tests are which? Choose two.

a) Minnesota Multiphasic Personality Inventory (MMPI)

b) Beck Depression Inventory

c) Frontal Assessment Battery (FAB)

d) Brief Psychiatric Rating Scale (BPRS)

e) Hamilton Rating Scale for Depression

 

The correct answers are:

a) Minnesota Multiphasic Personality Inventory (MMPI)

b) Beck Depression Inventory

Explanation:

  • The Minnesota Multiphasic Personality Inventory (MMPI) (a) is a psychological assessment tool that utilizes a questionnaire format to assess various personality traits and psychopathology.
  • The Beck Depression Inventory (b) is a self-report questionnaire designed to measure the severity of depression symptoms experienced by the individual.

25-5 (110B1) Which psychological and mental function test is conducted by presenting a predetermined set of 10 pictures in sequence?

a) Rorschach Test

b) Minnesota Multiphasic Personality Inventory (MMPI)

c) Wechsler Adult Intelligence Scale (WAIS-III)

d) Mini-Mental State Examination (MMSE)

e) Wisconsin Card Sorting Test (WCST)

 

The correct answer is:

a) Rorschach Test.

Explanation:

 

The Rorschach Test involves presenting a series of 10 predetermined inkblot cards to the individual being tested. The purpose of the test is to gain insight into the person's psychological state and thought processes based on their interpretations of the inkblots. It is unique among psychological tests in that it specifically uses a set of 10 inkblot pictures that are shown in a fixed order to elicit responses from the individual, which are then analyzed to assess psychological and mental functions. The other options listed do not involve a fixed sequence of 10 pictorial cards as part of their testing procedure.

25-6 (111G9) Which of the following psychological and mental function tests is not administered directly to the patient?

a) Tsunoda-Inage Developmental Test

b) Tanaka-Binet Intelligence Test

c) State-Trait Anxiety Inventory (STAI)

d) Minnesota Multiphasic Personality Inventory (MMPI)

e) Mini-Mental State Examination (MMSE)

 

The correct answer is:

a) Tsunoda-Inage Developmental Test

Explanation:

 

The Tsunoda-Inage Developmental Test, being focused on observing natural behaviors and responses in a structured setting or through caregiver reports, does not involve the direct, interactive testing typical of assessments.

25-7 (112F8) Which one is a self-report psychological test?

a) Rorschach Test

b) Tsunoda-Inage Developmental Test

c) State-Trait Anxiety Inventory (STAI)

d) Mini-Mental State Examination (MMSE)

e) Brief Psychiatric Rating Scale (BPRS)

 

The correct answer is:

c) State-Trait Anxiety Inventory (STAI)

Explanation:

 

The STAI is specifically designed as a self-report instrument, meaning that individuals fill out the questionnaire themselves, based on their own perceptions of their feelings and behaviors. It assesses two types of anxiety: state anxiety, or anxiety about an event; and trait anxiety, or anxiety level as a personal characteristic. Unlike the other options listed, which are either administered by a clinician (such as the Rorschach Test (a) and MMSE (d)) or observe behavior in a structured setting (like the Tsunoda-Inage Developmental Test (b)), or require a clinician's evaluation based on observations (as with the BPRS (e)), the STAI directly involves the individual's self-assessment regarding their anxiety levels, making it a distinct example of a self-report psychological test.

25- 8 (114C16) Which psychological or mental function test does the following question content belong to?

1) How old are you?

2) What is the date today (year, month, day)?

3) Where are we now?

4) Please repeat these three words I am going to say. Remember them because I will ask you to recall them later.

5) Subtract 7 from 100 in sequence.

6) Please repeat the numbers I say in reverse order.

7) Please recall the three words I asked you to remember earlier.

8) I will show you five items now. After hiding them, tell me what was there.

9) Name as many vegetables as you know.

a) Tanaka-Binet Intelligence Test

b) Rivermead Behavioural Memory Test

c) Wechsler Adult Intelligence Scale (WAIS-III)

d) Revised Hasegawa's Dementia Scale

e) Brief Psychiatric Rating Scale (BPRS)

 

The correct answer is:

d) Revised Hasegawa's Dementia Scale

Explanation:

 

The Revised Hasegawa's Dementia Scale (HDS-R) is a widely used cognitive screening tool for assessing dementia, particularly in older populations. This test includes questions designed to evaluate orientation to time and place (questions 1, 2, and 3), immediate recall (question 4), attention and calculation (question 5), working memory (question 6), delayed recall (question 7), and naming or recognition ability (question 8). Additionally, question 9, asking for as many items in a category as possible, assesses verbal fluency. These components are characteristic of tests aimed at evaluating cognitive functions broadly, with a specific emphasis on areas typically affected by dementia. Unlike the other options listed, which focus on different aspects of cognitive or psychological assessment, the HDS-R is specifically tailored to identify and assess the severity of dementia symptoms, making it the best match for the questions provided.

Clinical

26-1 (102A24) A 65-year-old woman was brought to the clinic by her family because she was causing a disturbance by claiming, "My daughter-in-law stole my purse." According to the family, the patient has recently become more selfish and irritable, and her forgetfulness has become more noticeable. Whenever the family searches for the purse with her, it is often found in a drawer of a desk. The patient, feeling resentful, says, "My daughter-in-law is mean."

Which test is useful for diagnosis?

a) Rorschach Test

b) Minnesota Multiphasic Personality Inventory (MMPI)

c) Mini-Mental State Examination (MMSE)

d) Brief Psychiatric Rating Scale (BPRS)

e) Hamilton Depression Rating Scale

 

The correct answer is:

c) Mini-Mental State Examination (MMSE)

Explanation:

 

The MMSE is a widely used tool for assessing cognitive function. It is particularly useful in screening for cognitive impairment and dementia. The scenario describes a 65-year-old woman exhibiting forgetfulness, changes in behavior (becoming more selfish and irritable), and accusations towards her daughter-in-law that appear unfounded, as the missing items are later found. These symptoms suggest possible cognitive impairment or the early stages of a dementia-related disorder. The MMSE can help evaluate the patient's orientation, recall ability, attention, calculation, language, and understanding to determine if there is a significant cognitive decline indicative of dementia or another cognitive disorder. The other tests listed (Rorschach Test, MMPI, BPRS, Hamilton Depression Rating Scale) assess different aspects of psychological functioning and are not as directly relevant to assessing the cognitive impairments implied by the patient's symptoms.

26-2 (109E51) A 30-year-old woman was brought to the clinic by her parents, who were concerned about her reclusive lifestyle. Around the age of 17, she began to say that people avoided her because she emitted a strange odor, leading her to confine herself to her room, after which she received psychiatric treatment. With treatment, she was able to go outside and attended a workshop. Around the age of 28, she began to experience auditory hallucinations, saying, "I am being gossiped about. Whatever I try to do, I am criticized," and ceased going out again, living a life of reading philosophy books over and over. Three months ago, she stopped visiting the clinic and taking her medication, prompting her parents to seek a new medical institution. During the examination, she showed a lack of emotional expression and passivity, with some fragmented hallucinatory and delusional complaints observed. No physical abnormalities were found.

Which psychological or mental function test is not useful for this patient?

a) Baum Test

b) Minnesota Multiphasic Personality Inventory (MMPI)

c) Mini-Mental State Examination (MMSE)

d) Wisconsin Card Sorting Test (WCST)

e) Brief Psychiatric Rating Scale (BPRS)

 

The correct answer is:

c) Mini-Mental State Examination (MMSE)

Explanation:

  • The MMSE is primarily a screening tool for cognitive impairment, focusing on areas such as orientation, memory, language abilities, and visuospatial skills. It is typically used to assess conditions like dementia or Alzheimer's disease. In the case of the 30-year-old woman described, the primary concerns are her reclusive lifestyle, the presence of auditory hallucinations, delusional thoughts, and a significant change in social behavior, which suggest a psychiatric disorder rather than cognitive decline. Her symptoms are more aligned with schizophrenia or a similar psychotic disorder, which involves issues like hallucinations, delusions, and withdrawal from reality, rather than the cognitive deficits the MMSE is designed to detect.
  • Other tests listed, such as the Baum Test (a projective test that can provide insights into an individual's personality structure) (a), the MMPI (which assesses personality traits and psychopathology) (b), the WCST (evaluating abstract thinking and the ability to shift problem-solving strategies) (d), and the BPRS (rating scale for psychiatric symptoms including depression and psychosis) (e), are more relevant for understanding her psychological state, personality structure, and specific symptoms. The MMSE's focus on cognitive functions such as memory and orientation does not directly address the psychological and psychiatric dimensions of her condition, making it the least useful option among those provided for assessing her mental and psychological functions.

26-3 (115C55) A 20-year-old woman visited the clinic primarily because she pulls out her hair and eyebrows. She was advised to consult psychiatry during a visit to internal medicine for headaches. Since the third grade of elementary school, pulling out her hair and eyebrows has become a habit, and currently, she has almost no hair and wears a wig. She had interviews with a school counselor, but it was interrupted after she started working. She has wanted to do something about it but has not had the courage to consult psychiatry until now. No appetite or sleep disorders are observed, and there is no significant impact on daily life. Her Hamilton Depression Rating Scale score is 12 points (0 to 7 points: normal).

Which two tests are appropriate for assessing this patient?

a) Rorschach Test

b) Tsunoda-Inage Developmental Test

c) Frontal Assessment Battery (FAB)

d) Sentence Completion Test (SCT)

e) Rivermead Behavioural Memory Test (RBMT)

 

The appropriate tests for assessing this patient are:

a) Rorschach Test

d) Sentence Completion Test (SCT)

Explanation:

  • The Rorschach Test can help uncover underlying emotional issues and thought patterns related to the patient's habit of pulling out her hair and eyebrows, a condition known as trichotillomania. This projective test can provide insights into the patient's inner experiences, conflicts, and personality structure that might be contributing to her behavior.
  • The Sentence Completion Test (SCT) is another useful tool in this context. It allows the patient to complete sentences in an open-ended manner, providing insights into her thoughts, feelings, and attitudes towards herself and her behavior. The SCT can reveal underlying psychological issues, such as stress, anxiety, or unresolved conflicts, which might be contributing to her compulsive behavior.
  • Both these tests are suited for a deeper psychological evaluation beyond the surface symptoms. They can help in understanding the psychological factors contributing to the patient's condition, facilitating a more targeted therapeutic approach. Other options, such as the Tsunoda-Inage Developmental Test (b), the Frontal Assessment Battery (FAB) (c), and the Rivermead Behavioural Memory Test (RBMT) (e), are less relevant to her symptoms, focusing more on cognitive development, frontal lobe functions, and memory, respectively, rather than underlying psychological issues.

26-4 (116C39) A 64-year-old woman was brought to the clinic by her family, who were concerned about her gradually worsening forgetfulness over the past year. Recently, she sometimes cannot remember if she has put away her wallet or had a meal.

Which test is useful for diagnosing this patient?

a) Rorschach Test

b) Tanaka-Binet Intelligence Test

c) Tsunoda-Inage Developmental Test

d) Wechsler Adult Intelligence Scale

e) Brief Psychiatric Rating Scale (BPRS)

 

The correct answer is:

d) Wechsler Adult Intelligence Scale

Explanation:

  • The Wechsler Adult Intelligence Scale (WAIS) is a comprehensive test designed to measure adult intelligence in several aspects, including verbal comprehension, perceptual reasoning, working memory, and processing speed. This test can help evaluate cognitive functions that are often affected by neurological conditions, such as dementia or Alzheimer's disease, which could be the cause of the patient's worsening forgetfulness and difficulty in remembering recent events, like putting away her wallet or having a meal.
  • The Rorschach Test (a) is a projective test used to assess underlying thought patterns and emotions but is not specifically designed to measure cognitive decline.
  • The Tanaka-Binet Intelligence Test (b) and the Tsunoda-Inage Developmental Test (c) are more focused on children's developmental stages and cognitive abilities, making them less suitable for assessing age-related cognitive decline in adults.
  • The Brief Psychiatric Rating Scale (BPRS) (e) assesses the severity of psychiatric symptoms rather than cognitive impairment directly related to memory and intellectual functioning.

General

27-1 (103A13) Which of the following are correct? Choose three.

a) Reserpine causes manic states.

b) Antihistamines cause restlessness.

c) Anti-Parkinson's drugs cause delirium.

d) Interferons cause depressive states.

e) Adrenocortical steroids cause mood alterations.

 

The correct answers are:

c) Anti-Parkinson's drugs cause delirium.

d) Interferons cause depressive states.

e) Adrenocortical steroids cause mood alterations.

Explanation:

  • c) Anti-Parkinson's drugs, which often increase dopamine activity, can lead to delirium, especially in older patients or those with pre-existing cognitive impairment. These medications aim to restore the balance of neurotransmitters in the brain but can sometimes overshoot, leading to confusion and disorientation.
  • d) Interferons, used in the treatment of certain cancers and viral infections, have been associated with the induction of depressive states in some patients. The exact mechanism isn't entirely understood but is thought to be related to interferon's effects on the immune system and brain chemistry.
  • e) Adrenocortical steroids, commonly used for their anti-inflammatory and immunosuppressive effects, can cause significant mood alterations, including euphoria, depression, and changes in personality. These effects are believed to result from steroids' wide-ranging impacts on the brain, including alterations in the metabolism of neurotransmitters.
  • a) Reserpine, which depletes neurotransmitters such as dopamine, serotonin, and norepinephrine, is more likely to cause depressive symptoms rather than manic states.
  • b) Antihistamines, particularly the first-generation ones, are more commonly associated with drowsiness rather than restlessness. Some of the newer, second-generation antihistamines are less likely to cause CNS side effects due to their inability to cross the blood-brain barrier.

27-2 (113D2) Which of the following is true about electroconvulsive therapy (ECT)?

a) It is not applicable for those over 65 years of age.

b) It is an applicable treatment for severe depression.

c) Muscle rigidity is seen as a side effect.

d) The presence of a neurosurgeon is required.

e) It can be conducted without the consent of the patient or their guardian.

 

The correct answer is:

b) It is an applicable treatment for severe depression.

Explanation:

  • Electroconvulsive therapy (ECT) is a well-established treatment modality, especially effective for severe depression that has not responded to other treatments. It involves the controlled induction of seizures in an anesthetized patient for therapeutic purposes. ECT has been shown to provide rapid and significant improvements in severe cases of depression, making it a valuable option in critical situations.
  • a) This statement is incorrect. ECT is not only applicable for individuals over 65 years of age but can be particularly effective in older adults, especially when there are risks associated with medication side effects or when rapid response is required.
  • c) Muscle rigidity is not a typical side effect of ECT. Patients are given muscle relaxants before the procedure to prevent muscle contractions during the seizure, thereby minimizing the risk of physical injury.
  • d) The presence of a neurosurgeon is not a requirement for ECT. The treatment team typically includes a psychiatrist, an anesthesiologist, and nursing staff. A neurosurgeon is not necessary for the procedure itself.
  • e) Consent is a critical component of ECT. It is ethically and legally required to obtain informed consent from the patient or their guardian (if the patient is not capable of giving consent) before proceeding with ECT, except in very specific emergency situations where the treatment is deemed lifesaving, and legal provisions exist to protect the patient's best interests.

27-3 (113F41) Which adverse events are likely to occur with benzodiazepine sleep medications? Choose two.

a) Falls

b) Aphasia

c) Intention tremor

d) Anterograde amnesia

e) Akathisia

 

The correct answers are:

a) Falls

d) Anterograde amnesia

Explanation:

  • a) Falls: Benzodiazepines can cause sedation, dizziness, and impair balance and coordination, increasing the risk of falls, especially in older adults. This risk is a significant concern, given the potential for serious injuries resulting from falls.
  • d) Anterograde amnesia: Benzodiazepines are known to affect memory, particularly the ability to form new memories. Anterograde amnesia refers to difficulty recalling events that occur after taking the medication, which is a well-documented effect associated with the use of benzodiazepines, especially in higher doses or when used for extended periods.
  • b) Aphasia, the loss or impairment of the ability to produce or comprehend language, is not typically associated with benzodiazepine use.
  • c) Intention tremor, a rhythmic, oscillating movement of a limb that is worse with movement or when trying to reach an object, is more commonly associated with neurological disorders like multiple sclerosis, not benzodiazepine use.
  • e) Akathisia, a state of restlessness and an inability to stay still, is more often associated with antipsychotic medications rather than benzodiazepines.

Clinical

28-1 (110I52) A 62-year-old man visited the clinic with a chief complaint of fever. He has had multiple admissions to psychiatric hospitals since about the age of 30 for schizophrenia and has been taking haloperidol, zotepine, and nitrazepam. Accompanied by his concerned family, he sought consultation due to a fever of 40°C and severe sweating starting yesterday. According to the family, he is less responsive than usual. His creatinine level from a health check-up last year was 0.7 mg/dL. Upon arrival, his consciousness level was JCS II-10. He is 168 cm tall and weighs 61 kg. His body temperature is 39.0°C, pulse 112/min, regular, and blood pressure 150/82 mmHg. There is no erythema of the pharyngeal mucosa, and no abnormalities in the chest. Bowel sounds are decreased. Strong muscle rigidity is observed. Urinalysis shows protein 1+, blood 2+, with 1-4 red blood cells per high power field in the sediment. Blood tests show: red blood cells 3.04 million, Hb 9.5 g/dL, Ht 27%, white blood cells 8,800, platelets 130,000. Biochemical tests show: total protein 6.5 g/dL, albumin 3.6 g/dL, AST 225 IU/L, ALT 129 IU/L, LD 848 IU/L (normal 176-353), CK 35,000 IU/L (normal 30-140), blood urea nitrogen 53 mg/dL, creatinine 2.5 mg/dL, Na 135 mEq/L, K 5.3 mEq/L, Cl 106 mEq/L.

What is the appropriate response?

a) Administration of immunoglobulin

b) Steroid pulse therapy

c) Continuation of antipsychotic medication

d) Red blood cell transfusion

e) Aggressive fluid resuscitation

 

The appropriate response is:

e) Aggressive fluid resuscitation

Explanation:

 

The patient's presentation, including high fever, severe sweating, muscle rigidity, and altered consciousness, along with the laboratory findings of elevated creatinine kinase (CK) levels, suggests a diagnosis of Neuroleptic Malignant Syndrome (NMS). NMS is a life-threatening condition associated with the use of antipsychotic medications, characterized by hyperthermia, altered mental status, muscle rigidity, and autonomic dysfunction.

  • Aggressive fluid resuscitation is crucial in the management of NMS to address dehydration and help in the elimination of myoglobin released from damaged muscle cells, thereby preventing renal failure. The high CK level indicates rhabdomyolysis, a breakdown of muscle tissue leading to the release of muscle fiber contents into the blood, which can be harmful to the kidneys.
  • Other options, such as the administration of immunoglobulin (a) and steroid pulse therapy (b), are not standard treatments for NMS. Continuation of antipsychotic medication (c) would be contraindicated in this situation as antipsychotics are the causative agents of NMS. Red blood cell transfusion (d) might be considered in cases of severe anemia, but the immediate concern in NMS is addressing hyperthermia, autonomic instability, and preventing renal damage through adequate hydration and supportive care.

28-2 (115A24) A 55-year-old man visited the clinic with his wife, complaining of having no memory of events that occur during the night. He had been prescribed sleeping pills for insomnia a few years ago and has continued to take them while maintaining his job. He appeared to be severely depressed after his restaurant was burglarized two weeks ago. Yesterday, he returned home at 7 PM, finished dinner, and went to bed at 11 PM. At about 1 AM the next day, a friend, trying to cheer him up, called to invite him to karaoke. He took a taxi to the karaoke place, participated in the party, and returned home around 4 AM. He slept for about 8 hours before going to his afternoon shift but could not remember anything about the night. According to his friend, he sang and ate normally. He cannot drink alcohol at all and did not drink any that day. His wife mentioned that about two months ago, there were three instances where he ate during the night or went to a convenience store and could not remember doing so the next morning.

Which condition is considered in this patient?

a) Nocturnal delirium

b) Transient global amnesia

c) Lifelong amnesia

d) Anterograde amnesia due to sleeping pills

e) REM sleep behavior disorder

 

The condition considered in this patient is:

d) Anterograde amnesia due to sleeping pills

Explanation:

  • Anterograde amnesia refers to the inability to form new memories after the onset of amnesia, meaning that the person cannot remember new information for a certain period. This condition is closely associated with the use of certain sleeping pills, especially benzodiazepines and non-benzodiazepine sleep aids, which are known to affect memory formation. The patient's history of continuous sleeping pill use, coupled with episodes of engaging in activities at night (like going to karaoke and visiting convenience stores) without any memory of them the next day, strongly suggests anterograde amnesia caused by his medication.
  • Nocturnal delirium (a) typically involves confusion and agitation during the night, which is not described here. Transient global amnesia (b) is a sudden, temporary episode of memory loss that is more acute and not linked to sleeping pill use. Lifelong amnesia (c) would involve a more extended period of memory loss that spans years, not just recent events. REM sleep behavior disorder (e) involves physically acting out vivid dreams with vocal sounds and sudden, often violent arm and leg movements during REM sleep, which does not match the patient's symptoms of simply not remembering his actions.

28-3 (116D70) A 66-year-old woman visited the clinic complaining of insomnia. She has had trouble falling asleep and wakes up two to three times during the night since three months ago. No cognitive decline or feelings of depression were observed, and her appetite was normal, so she was prescribed a benzodiazepine sleeping pill.

Which of the following are appropriate explanations for the patient? Choose two.

a) "You may become more sensitive to cold."

b) "You may experience unsteadiness on your feet."

c) "Your gums may become swollen."

d) "You may experience breast tenderness and milk secretion."

e) "If you suddenly stop taking it after continuous use, you may experience anxiety."

 

The appropriate explanations for the patient are:

b) "You may experience unsteadiness on your feet."

e) "If you suddenly stop taking it after continuous use, you may experience anxiety."

Explanation:

  • b) Benzodiazepines can affect muscle coordination and balance, making unsteadiness or a feeling of being off-balance a possible side effect, especially in older adults. This risk is important to communicate, as it can increase the risk of falls.
  • e) Benzodiazepines can lead to physical dependence, and abrupt discontinuation after prolonged use can cause withdrawal symptoms, including increased anxiety, restlessness, and insomnia. It's crucial for patients to be aware of this risk so they can avoid suddenly stopping the medication without medical guidance.

The other options are not typically associated with benzodiazepine use:

  • a) Increased sensitivity to cold is not a common side effect of benzodiazepines.
  • c) Swollen gums are not a recognized side effect of benzodiazepine sleeping pills.
  • d) Breast tenderness and milk secretion are side effects associated with drugs that influence prolactin levels, not typically with benzodiazepines.